OB PrepU: Chapter 20, OB PrepU: Chapter 19, OB PrepU: Chapter 16

Réussis tes devoirs et examens dès maintenant avec Quizwiz!

The nurse can expect a patient who had a cesarean birth to have less lochia discharge than the patient who had a vaginal birth. a) True b) False

a) True Rationale: Women who had a cesarean birth will have less lochia discharge than those who had a vaginal birth, but stages and color changes remain the same.

You are caring for a patient with preeclampsia. You know that you need to auscultate this patient's lung sounds every two hours. Why would you do this? a) Pulmonary emboli b) Pulmonary edema c) Pulmonary atelectasis d) Pulmonary hypertension

b) Pulmonary edema Rationale: In the hospital, monitor blood pressure at least every four hours for mild preeclampsia and more frequently for severe disease. In addition, it is important to auscultate the lungs every two hours. Adventitious sounds may indicate, developing pulmonary edema.

Which factor would contribute to a high-risk pregnancy? a) First pregnancy at age 33 b) Type 1 diabetes c) History of allergy to honey bee pollen d) Blood type O positive

b) Type 1 diabetes Rationale: A woman with a history of diabetes has an increased risk for perinatal complications, including hypertension, preeclampsia, and neonatal hypoglycemia. The age of 33 without other risk factors doesn't increase risk, nor does type O-positive blood or environmental allergens.

Which medication is prescribed most commonly for a pregnant woman with chronic hypertension? a) Nifedipine b) Atenolol c) Methyldopa d) Labetolol

c) Methyldopa Rationale: Although labetolol, atenolol, and nifedipine may be ordered, methyldopa the most commonly prescribed agent because of its safety record during pregnancy. It is a slow-acting antihypertensive agent that also helps to improve uterine perfusion.

Your pregnant patient has had asthma since she was a teenager. What statement by the patient would alert you to the fact her asthma may not be in control? a) Daytime cough b) Decreased respiratory rate c) Feeling of euphoria d) "I keep waking up at night."

d) "I keep waking up at night." Rationale: Complaint of nocturnal awakening is a classic symptom.

An alert, diabetic, pregnant woman in the hospital experiences some shakiness and diaphoresis with a fasting blood sugar of 60 mg/dl when she awakens in the morning. Which action should the nurse take first? a) Recheck her blood sugar for accuracy. b) Stay with her and ask another nurse to bring her insulin. c) Withhold her insulin and notify the health care provider. d) Administer the patient's glucose tablets.

d) Administer the patient's glucose tablets. Rationale: The patient is hypoglycemic when awakening in the morning. The nurse should provide glucose and be prepared to reassess. The nurse should not recheck at this point, since the patient is symptomatic. She does not need insulin, and she will have her morning dose adjusted after breakfast.

A woman who has sickle cell anemia asks you if her infant will develop sickle cell disease. The nurse would base the answer on which of the following? a) Sickle cell anemia is not inherited; it occurs following a malaria infection. b) Sickle cell anemia is dominantly inherited. c) Sickle cell anemia has more than one polygenic inheritance pattern. d) Sickle cell anemia is recessively inherited.

d) Sickle cell anemia is recessively inherited. Rationale: Sickle cell anemia is an autosomal recessive disease requiring that the person have two genes for the disease, one from each parent. If one parent has the disease and the other is free of the disease and trait, the chances of the child inheriting the disease is zero. Options B, C, and D are incorrect as they give misinformation to the patient and are inappropriate answers for the nurse to give.

A pregnant woman with type 2 diabetes is scheduled for a laboratory test of glycosylated hemoglobin (HbA). What does the nurse tell the patient is a normal level for this test? a) 6% b) 14% c) 8% d) 12%

a) 6% Rationale: The upper normal level of HbA is 6% of total hemoglobin.

A client experiences a threatened abortion. She is concerned about losing the pregnancy and asks what activity level she should maintain. What is the most appropriate response from the nurse? a) "Restrict your physical activity to moderate bedrest." b) "There is no research evidence that I can recommend to you." c) "Carry on with the activity you engaged in before this happened." d) "Strict bedrest is necessary so as not to jeopardize this pregnancy."

a) "Restrict your physical activity to moderate bedrest." Rationale: With a threatened abortion, moderate bedrest and supportive care are recommended. Regular physical activity may increase the chances of miscarriage. Strict bedrest is not necessary. Activity restrictions are part of standard medical management.

A nursing instructor is teaching students about anemia during pregnancy. Which type of anemia does the instructor teach students is most prevalent during pregnancy? a) sickle-cell anemia b) pernicious anemia c) iron-deficiency anemia d) folic acid anemia

c) iron-deficiency anemia Rationale: Iron-deficiency anemia is the most common type in pregnancy. Many woman enter pregnancy with a low iron count because of poor diet, heavy menstrual periods, unwise weight-loss programs, or a combination of these.

Spontaneous miscarriage occurs in 5% to 10% of all pregnancies. a) False b) True

a) False Rationale: Spontaneous miscarriage occurs in 15% to 30% of all pregnancies and arises from natural causes.

The nurse is assessing a woman with class III heart disease who is in for a prenatal visit. What would be the first recognizable sign that this patient is in heart failure? a) Elevated blood pressure b) Low blood pressure c) Audible wheezes d) Persistent rales in the bases of the lungs

d) Persistent rales in the bases of the lungs Rationale: The earliest warning sign of cardiac decompensation is persistent rales in the bases of the lungs.

A pregnant woman is determined to be at high risk for gestational diabetes. At which time would the nurse expect the client to undergo rescreening? a) 28 to 32 weeks b) 16 to 20 weeks c) 20 to 24 weeks d) 24 to 28 weeks

d) 24 to 28 weeks Rationale: A woman identified as high risk would undergo rescreening between 24 and 28 weeks.

Postpartum bleeding must be assessed carefully during the first 24 hours after delivery. Prioritize the actions taken upon detection of increased vaginal bleeding in a patient who delivered within the last 24 hours.

1. Palpate the fundus 2. Massage the fundus if boggy 3. Notify the physician or the nurse-midwife of excessive bleeding 4. Increase IV pitocin or breastfeed the newborn 5. Assess blood pressure 6. Assist the patient to empty her bladder in the bathroom Rationale: Determining the site of bleeding is the first assessment. Palpate the fundus. If the fundus is boggy, take steps to stimulate contractions by massaging. Notify the health care provider, and continue with the assessment. Stimulate contractions. Assess blood pressure and assess for safety to ambulate. A likely reason for a boggy uterus is dislocation by a full bladder.

Which of the following would the nurse include when teaching the parents of a newborn who have a 2-year-old boy at home? a) "Ask your 2-year-old to pick out a special toy for his sister." b) "Talk to your 2-year-old about the baby when you're driving him to day care." c) "Have your 2-year-old stay at home while you're here in the hospital." d) "Expect to see your 2-year-old become more independent when the baby gets home."

a) "Ask your 2-year-old to pick out a special toy for his sister." Rationale: The parents should encourage the sibling to participate in some of the decisions about the baby, such as names or toys. Typically siblings experience some regression with the birth of a new baby. The parents should talk to the sibling during relaxed family times. The parents should arrange for the sibling to come to the hospital to see the newborn.

Vaginal bleeding during pregnancy is always a deviation from the normal. a) True b) False

a) True Rationale: Vaginal bleeding during pregnancy is always a deviation from the normal, is always potentially serious, may occur at any point during pregnancy, and is always frightening. It must always be carefully investigated because it can impair both the outcome of the pregnancy and the woman's life or health.

A woman with cardiac disease delivered a seven pound baby by C-Section. Which of the following interventions should be implemented during the immediate postpartum period? a) Limit visits with the infant so mom may rest. b) Ambulate to bathroom only. c) Rest, stool softeners, and monitoring tolerance of activity. d) Restrict activity to bedrest.

c) Rest, stool softeners, and monitoring tolerance of activity. Rationale: A woman who has a cardiac condition is at increased risk in the postpartum period. She needs frequent assessment and observation for tolerance. She would also be given education to avoid straining activities such as bowel movements and would be encouraged to have stool softeners and increase fluid and fiber. Restricting the patient's activity to bed rest could be detrimental to the patient, as could be ambulating to the bathroom only. There is no reason to limit the visits with the infant.

A nurse is instructing a woman that it is important to lose pregnancy weight gain within 6 months of delivery, because studies show that keeping extra weight longer is a predictor of which of the following? a) diabetes b) feelings of increased self-esteem c) increased sex drive d) long-term obesity

d) long-term obesity Rationale: Women who have not returned to their prepregnant weight by 6 months postpartum are likely to retain extra weight. This inability to lose is a predictor of long-term obesity. It will not necessarily lead to diabetes, but it may decrease a woman's self-esteem and sex drive if she feels less attractive with the extra weight.

A client at 11 weeks' gestation experiences pregnancy loss. The client asks the nurse if the bleeding and cramping that occurred during the miscarriage were caused by working long hours in a stressful environment. What is the most appropriate response from the nurse? a) "I can understand your need to find an answer to what caused this. Let's talk about this further." b) "It is hard to know why a woman bleeds during early pregnancy." c) "Your spontaneous bleeding is not work-related." d) "Something was wrong with the fetus."

a) "I can understand your need to find an answer to what caused this. Let's talk about this further." Rationale: Talking with the client may assist her to explore her feelings. She and her family may search for a cause for a spontaneous early bleeding so they can plan for future pregnancies. Even with modern technology and medical advances, however, a direct cause cannot usually be determined.

Fourteen-year-old Ann and her parents have presented at the obstetrician's office in the second trimester, the teen had been hiding the pregnancy. The nurse is helping them develop a plan of care. What is the best thing she can say to the clearly angry parents? a) "I know you must be very upset and angry about Ann's pregnancy but because she's still an adolescent herself, she'll need your guidance in making nutritional and health choices that will be good for the baby and for herself." b) "Anger won't help this situation at all. You'll only push Ann away and she'll be less likely to make good choices." c) "Ann needs to make decisions about this pregnancy for herself." d) "I understand your anger but if you had encouraged Ann to use condoms she would probably not be in this situation."

a) "I know you must be very upset and angry about Ann's pregnancy but because she's still an adolescent herself, she'll need your guidance in making nutritional and health choices that will be good for the baby and for herself." Rationale: The nurse needs to acknowledge the anger of the parents but remember her role is as the patient advocate. The nurse needs to encourage the relationship of support between the parents and the patient. Option A is incorrect as it is attempting to lay down ground rules between the patient and her parents, it also does not acknowledge the parents' feelings in this situation. Option C is incorrect as it might be interpreted as a lecture to the parents, and it does not acknowledge their feelings in this situation. Option D is incorrect; again it sounds like lecturing and it places blame on the parents, which is inappropriate.

A student nurse asks the instructor what percentage of clinically recognized pregnancies end in miscarriages during the first trimester. The most accurate response from the registered nurse is which of the following? a) 11% to 20% b) 21% to 30% c) 5% to 10% d) 31% to 40%

a) 11% to 20% Rationale: During the first trimester, 10% to 20% of all clinically recognized pregnancies end in miscarriage.

Which finding would the nurse describe as "light" or "small" lochia? a) 4-inch stain or a 1 to 25 ml loss b) 1- to 2-inch lochia stain on the perineal pad or a 10 ml loss c) 4- to 6-inch stain with an estimated loss of 25 to 50 ml d) pad is saturated within 1 hour after changing it

a) 4-inch stain or a 1 to 25 ml loss Rationale: Typically the amount of lochia is described as follows: Scant: a 1- to 2-inch lochia stain on the pad or a 10 ml loss; Light or small: 4-inch stain or a 10 to 25 ml loss; Moderate: 4- to 6-inch stain with an estimated loss of 25 to 50 ml; Large or heavy: a pad is saturated within 1 hour after changing it.

A primipara at 36 weeks gestation is being monitored in the prenatal clinic for risk of preeclampsia. Which of the following signs or symptoms is the priority concern for the nurse? a) A dipstick value of 2+ for protein. b) Weight gain of 1.2 lb during the past 1 week. c) A systolic blood pressure increase of 10 mm hg. d) Pedal edema.

a) A dipstick value of 2+ for protein. Rationale: The increasing amount of protein in the urine is a concern the preeclampsia may be progressing to severe preeclampsia. The woman needs further assessment by the health care provider. Dependent edema may be seen in the majority of pregnant women and is not an indicator of progression from preeclampsia to eclampsia. Weight gain is no longer considered an indicator for the progression of preeclampsia. A systolic blood pressure increase is not the highest priority concern for the nurse, since we have no idea what the baseline blood pressure was.

Patient teaching is conducted throughout a patient's hospitalization and is reinforced before discharge. Which self-care items are to be reinforced before discharge? a) Activity b) Infant formula selection c) Resumption of intercourse d) Signs and symptoms of infection e) Resumption of prepregnancy diet

a) Activity c) Resumption of intercourse d) Signs and symptoms of infection Rationale: The correct answers give information on managing changes in her new role as a mother. The assumption cannot be made that her prepregnancy diet is still appropriate, and the formula choice should be discussed with her pediatrician.

A pregnant client is admitted to a health care unit with disseminated intravascular coagulation (DIC). Which of the following orders is the nurse most likely to receive regarding the therapy for such a client? a) Administer cryoprecipitate and platelets b) Give each unit of blood to raise the hematocrit by 3 g/dL c) Administer a ratio of 1 unit of blood to 4 units of frozen plasma d) Aim at keeping the client's hematocrit above 20%

a) Administer cryoprecipitate and platelets. Rationale: In a pregnant client with DIC, the nurse may be ordered to administer cryoprecipitate and platelets. Whole blood does not contain clotting factors. Therefore a ratio of 4 units of blood to 1 unit of fresh frozen plasma, and not 1 unit of blood to 4 units of frozen plasma, should be considered. The nurse should aim at maintaining the client's hematocrit above 30% and not just 20%. The nurse should expect one unit of blood to increase the hematocrit by 1.5 g/dL, and not 3g/dL.

A client with juvenile rheumatoid arthritis (JRA) is in week 38 of her pregnancy. Which of the following interventions should the nurse make with this client? a) Ask the client to decrease her intake of salicylates b) Perform the Snellen eye test c) Urge the client to be on bed rest d) Advise the client to continue her normal dosage of methotrexate

a) Ask the client to decrease her intake of salicylates Rationale: Although women with JRA should continue to take their medications during pregnancy to prevent joint damage, large amounts of salicylates have the potential to lead to increased bleeding at birth or prolonged pregnancy. The infant may be born with a bleeding defect and may also experience premature closure of the ductus arteriosus because of the drug's effects. For this reason, a woman is asked to decrease her intake of salicylates approximately 2 weeks before term. A number of women also take low-dose methotrexate, a carcinogen. As a rule, they should stop taking this prepregnancy because of the danger of head and neck defects in the fetus. There is no need for the client to be on bed rest or to perform the Snellen eye test.

When caring for a postpartum woman who is Muslim, which of the following would be a priority? a) Assigning a female nurse to care for her b) Allowing time for the numerous visitors who come to see the woman and newborn c) Ensuring that the newborn's daily bath is performed by the nurses d) Providing time for prayers to be performed at the bedside

a) Assigning a female nurse to care for her Rationale: Muslims prefer the same-sex health care provider; male-female touching is prohibited except in emergency situations. Nurses give the daily bath for newborns of some Japanese-American women. Numerous visitors can be expected to visit some women of the Filipino-American culture because families are very closely knit. Bedside prayer is common due to the strong religious beliefs of the Filipino-American culture.

A G1 P1001 mother is just home after delivering her first child 5 days ago. Her delivery was complicated by an emergency cesarean delivery resulting from incomplete cervical dilation and hemorrhage. The nurse determines that the mother has not slept longer than 3 hours at one time. The appropriate nursing diagnosis for this patient care issue is a) At risk for postpartum depression due to inadequate rest b) At risk for interruption of tissue integrity c) At risk for safety due to low hemoglobin d) At risk for inadequate healing due to decreased nutrition

a) At risk for postpartum depression due to inadequate rest Rationale: This scenario refers only to the issue of sleep. Information is insufficient to suggest that the other issues are problematic at this time.

A patient is admitted at 22 weeks gestation with advanced cervical dilatation to 5 centimeters, cervical insufficiency, and a visible amniotic sac at the cervical opening. What is the primary goal for this patient at this point? a) Bed rest to maintain pregnancy as long as possible b) Deliver vaginally c) Notification of social support for loss of pregnancy d) Education on causes of cervical insufficiency for the future

a) Bed rest to maintain pregnancy as long as possible Rationale: At 22 weeks gestation, the fetus is not viable. The woman would be placed on bed rest, total, with every attempt made to halt any further progression of dilatation as long as possible. You would not want to deliver this fetus vaginally at this stage of gestation. It is not your responsibility to notify the patient's social support of a possible loss of the pregnancy. It is not appropriate at this time to educate the mother on causes of cervical insufficiency for future pregnancies.

A woman in week 40 of her pregnancy has developed a urinary tract infection (UTI). The nurse recognizes that which of the following treatments would be safe and appropriate to use with this client? (Select all that apply.) a) Cephalosporins b) Heparin c) Amoxicillin d) Sulfonamides e) Tetracyclines f) Ampicillin

a) Cephalosporins c) Amoxicillin f) Ampicillin Rationale: Amoxicillin, ampicillin, and cephalosporins are effective against most organisms causing UTIs and are safe antibiotics during pregnancy. The sulfonamides can be used early in pregnancy but not near term because they can interfere with protein binding of bilirubin, which then leads to hyperbilirubinemia in the newborn. Tetracyclines are contraindicated during pregnancy as they cause retardation of bone growth and staining of the fetal teeth. Heparin is an anticoagulant and is used to prevent clot formation; it would not be prescribed for a UTI.

A woman with a positive history of genital herpes is in active labor. She has small pin-point vesicles in the perineum area. Her membranes are ruptured, she is dilated 5cm, effaced 70%. The nurse should anticipate what type of delivery? a) Cesarean. b) Forceps assisted. c) Spontaneous vaginal. d) Vacuum assisted.

a) Cesarean. Rationale: An active herpes infection can be passed to the fetus during labor or with ruptured amniotic membranes. The nurse should anticipate the infant will be delivered via a cesarean birth. The risk of transmitting herpes to the baby would be increased if the baby were born by spontaneous vaginal delivery, vacuum assisted delivery, or forceps assisted delivery.

You are caring for a young woman who is in her 10th week of gestation. She comes into the clinic complaining of vaginal bleeding. Which assessment finding best correlates with a diagnosis of hydatidiform mole? a) Dark red, "clumpy" vaginal discharge b) Painful uterine contractions and nausea c) Bright red painless vaginal bleeding d) Brisk deep tendon reflexes and shoulder pain

a) Dark red, "clumpy" vaginal discharge Rationale: If a complete molar pregnancy continues into the second trimester undetected, other signs and symptoms appear. The woman often presents with complaints of dark to bright red vaginal bleeding and pelvic pain. Infrequently, she will report passage of grapelike vesicles.

A client reports to her obstetrician complaining of a significant amount of bright red, painless vaginal bleeding. A sonogram reveals that her placenta has implanted low in the uterus and is partially covering the cervical os. Which of the following immediate care measures should the nurse initiate? (Select all that apply.) a) Determine from the client the time the bleeding began and about how much blood has been lost b) Attach external monitoring equipment to record fetal heart sounds c) Continue to assess blood pressure every 5 to 15 minutes d) Perform a pelvic examination e) Place the woman on bed rest in a side-lying position f) Obtain baseline vital signs

a) Determine from the client the time the bleeding began and about how much blood has been lost b) Attach external monitoring equipment to record fetal heart sounds c) Continue to assess blood pressure every 5 to 15 minutes e) Place the woman on bed rest in a side-lying position f) Obtain baseline vital signs Rationale: With the exception of performing a pelvic examination, all of the answers are appropriate immediate care measures for the client with placenta previa. Never attempt a pelvic or rectal examination with painless bleeding late in pregnancy because any agitation of the cervix when there is a placenta previa might tear the placenta further and initiate massive hemorrhage, possibly fatal to both mother and child.

A 29-year-old client has gestational diabetes. The nurse is teaching her about managing her glucose levels. Which therapy would be most appropriate for this client? a) Diet b) Glucagon c) Long-acting insulin d) Oral hypoglycemic drugs

a) Diet Rationale: Clients with gestational diabetes are usually managed by diet alone to control their glucose intolerance. Long-acting insulin usually isn't needed for blood glucose control in the client with gestational diabetes. Oral hypoglycemic drugs are contraindicated in pregnancy. Glucagon raises blood glucose and is used to treat hypoglycemic reactions.

A client in her 20th week of gestation develops HELLP syndrome. Which of the following should the nurse consider as features of HELLP syndrome? Select all that apply. a) Elevated liver enzymes b) Leukocytosis c) Low platelet count d) Hemolysis e) Hyperthermia

a) Elevated liver enzymes c) Low platelet count d) Hemolysis Rationale: The HELLP syndrome is a syndrome involving hemolysis (microangiopathic hemolytic anemia), elevated liver enzymes, and a low platelet count. Hyperthermia and leukocytosis are not features of HELLP syndrome.

A woman with an incomplete abortion is to receive misoprostol. The nurse understands that the rationale for administering this drug is to: a) Ensure passage of all the products of conception b) Suppress the immune response to prevent isoimmunization c) Halt the progression of the abortion d) Alleviate strong uterine cramping

a) Ensure passage of all the products of conception Rationale: Misoprostol is used to stimulate uterine contractions and evacuate the uterus after an abortion to ensure passage of all the products of conception. Rh (D) immunoglobulin is used to suppress the immune response and prevent isoimmunization.

The nurse should encourage a pregnant client who is taking short-acting insulin for her diabetes to avoid eating after self-administering the insulin. a) False b) True

a) False Rationale: Caution women with diabetes to eat almost immediately after injecting short-acting insulin to prevent hypoglycemia before mealtimes.

A pregnant woman with sickle cell anemia comes to the emergency department in crisis. Which of the following would the nurse expect to find? Select all that apply. a) Fever b) Pallor c) Increased skin turgor d) Joint pain e) Fatigue

a) Fever d) Joint pain Rationale: Signs and symptoms of a sickle cell crisis commonly include severe abdominal pain, muscle spasm, leg pains, joint pain, fever, stiff neck, nausea and vomiting, and seizures. Skin turgor would most likely be poor because the client would probably be dehydrated. Pallor and fatigue are associated with sickle cell anemia and would not help identify a crisis.

You are the clinic nurse caring for a pregnant woman in her third trimester. The woman is HIV positive and voices concerns about passing the infection on to her baby. What is your best response? a) If you are taking antiretroviral medications and you don't breastfeed your baby, you greatly reduce the risk of perinatal transmission of the disease. b) Perinatal transmission of the virus is a real fear. Would you like to talk to a social worker? c) There is nothing you can do. You will just have to wait and see if your baby is born HIV positive. d) Do you have other children? Your baby has a one-in-four chance of having HIV at birth, so if you have three other children who are not HIV positive, then this one will be HIV positive.

a) If you are taking antiretroviral medications and you don't breastfeed your baby, you greatly reduce the risk of perinatal transmission of the disease. Rationale: Receiving appropriate antiretroviral treatment during pregnancy and childbirth and refraining from breastfeeding substantially reduce the risk of perinatal transmission.

A 16-year-old girl comes to the public health office and tells you she is pregnant. She is afraid to tell her parents. As a nurse, what is important for you to know that can help this 16 year old? a) Know about community resources for the pregnant teen b) Know who the father of the baby is c) Know who the mother's parents are d) Know what school district she resides in

a) Know about community resources for the pregnant teen Rationale: Be knowledgeable regarding community resources for the pregnant teen. If you or the primary-care practitioner refers the teen to another entity, follow up to make certain the adolescent receives the services for which she was referred. If she does not, try to determine the barriers that prevent her from following through with treatment. Assist her to work through the barriers to obtain needed services.

A pregnant woman at 4 weeks' gestation who has preexisting diabetes mellitus visits her primary care provider for a check-up. Which fetal complications might occur because of this maternal condition? Select all that apply. a) Macrosomia (oversized fetus) b) Respiratory disorder c) Congenital malformations d) Fetus with juvenile diabetes e) Smaller than gestational age baby

a) Macrosomia (oversized fetus) b) Respiratory disorder c) Congenital malformations Rationale: Potential problems during pregnancy involving maternal diabetes mellitus include fetal death, macrosomia (oversized fetus), a fetus with a respiratory disorder, difficult labor, preeclampsia or eclampsia, polyhydramnios, and congenital malformations.

Two days ago, a woman delivered her third infant; she is now preparing for discharge home. After the delivery of her second child, she developed an endometrial infection. Nursing goals for this discharge include all of the following EXCEPT a) Maintain previous household routines to prevent infection b) The patient will show no signs of infection c) Discuss methods that the woman will use to prevent infection d) List signs of infection that she will report to her health care provider

a) Maintain previous household routines to prevent infection Rationale: The nurse does not know whether previous routines were or were not the source of the infection. The other three options provide correct instructions to be given to this woman.

Which of the following would the nurse most likely include when planning the care for a woman requiring hospitalization for hyperemesis gravidarum? Select all that apply. a) Maintaining NPO status for the first day or two b) Administering antiemetic agents c) Preparing the woman for insertion of a feeding tube d) Obtaining baseline blood electrolyte levels e) Monitoring intake and output

a) Maintaining NPO status for the first day or two b) Administering antiemetic agents d) Obtaining baseline blood electrolyte levels e) Monitoring intake and output Rationale: When hospitalization is necessary, oral food and fluids are withheld to allow the gut to rest. Antiemetic agents are ordered to help control nausea and vomiting. The woman is likely to be dehydrated, so the nurse would obtain baseline blood electrolyte levels and administer intravenous fluid and electrolyte replacement therapy as indicated. Once the nausea and vomiting subside, oral food and fluids are gradually reintroduced. Total parenteral nutrition or a feeding tube is used to prevent malnutrition only if the client does not improve with these interventions.

A nurse assessing a postpartum patient notices excessive bleeding. What should be the nurse's first action? a) Massage the boggy fundus until it is firm. b) Document the findings. c) Call the physician. d) Nothing--excessive postpartum blood loss is normal.

a) Massage the boggy fundus until it is firm. Rationale: The nurse needs to report any abnormal findings when assessing the lochia. If excessive bleeding occurs, the first step would be to massage the boggy fundus until it is firm to reduce the flow of blood. Then the nurse needs to document the findings.

Given that the first 24 hours after delivery is a time for return to homeostasis, which postpartum findings are considered acceptable during this time? Select all that apply. a) Moderate saturation of peripad every 3 hours b) Fundus one fingerbreadth above umbilicus c) Hypotonic bowel sounds d) Inverted nipples following breastfeeding e) Urination of 50 mL every hour

a) Moderate saturation of peripad every 3 hours b) Fundus one fingerbreadth above umbilicus Rationale: A fundus can rise to slightly above or below the umbilicus in the first 24 hours, and moderate saturation of 2/3 of the pad is appropriate. Inverted nipples always require intervention if breastfeeding. Hypotonic bowel sounds also require assessment more frequently than routinely ordered, and 50 mL urine is inadequate given the occurrence of diuresis.

A pregnant client has been admitted with complaints of brownish vaginal bleeding. On examination there is an elevated hCG level, absent fetal heart sounds and a discrepancy between the uterine size and the gestational age. The nurse interprets these findings to suggest which of the following? a) Molar pregnancy b) Abruption of placenta c) Placenta previa d) Ectopic pregnancy

a) Molar pregnancy Rationale: The client is most likely experiencing molar pregnancy. In molar pregnancy, there is an abnormal proliferation and eventual degeneration of the trophoblastic villi. The signs and symptoms of molar pregnancy include brownish vaginal bleeding, elevated hCG levels, discrepancy between the uterine size and the gestational age, and absent fetal heart sounds. Abruption of placenta is characterized by premature separation of the placenta. Ectopic pregnancy is a condition where there is implantation of the blastocyst outside the uterus. In placenta previa the placental attachment is at the lower uterine segment.

A 24-year-old woman presents at the emergency room with abdominal pain, vaginal bleeding, and fatigue. What should the nurse's first action be? a) Monitor the patient for signs of shock. b) Ascertain from the patient if she has been sexually active to rule out pregnancy. c) Prepare for IV blood transfusion. d) Assume a pregnancy and prepare to determine the viability of the fetus.

a) Monitor the patient for signs of shock. Rationale: Anytime a patient presents with hemorrhage, the initial nursing consideration is assessment and evaluation of shock. This is also the case in the pregnant patient. If the mother is in shock the fetus will have lack of oxygen. Monitor the maternal status first. Ascertaining sexual activity to rule out pregnancy or assuming a pregnancy and preparing to determine the viability of the fetus is not the initial nursing consideration for this patient. The scenario does not indicate the need for a blood transfusion.

A pregnant woman diagnosed with diabetes should be instructed to do which of the following? a) Notify the physician if unable to eat because of nausea and vomiting. b) Prepare foods with increased carbohydrates to provide needed calories. c) Ingest a smaller amount of food prior to sleep to prevent nocturnal hyperglycemia. d) Discontinue insulin injections until 15 weeks gestation.

a) Notify the physician if unable to eat because of nausea and vomiting. Rationale: During pregnancy, the insulin levels change in response to the production of HPL. The patient needs to alert her provider if she is not able to eat or hold down appropriate amounts of nutrition. The patient is at risk for episodes of hypoglycemia during the first trimester. She should never discontinue insulin therapy without her provider's directions. The increase of carbohydrates needs to be balanced with protein, and smaller meals would result in hypoglycemia rather than hyperglycemia.

A nurse is assessing a client in her seventh month of pregnancy who has an artificial valve prosthesis. The client is taking an oral anticoagulant to prevent the formation of clots at the valve site. Which of the following nursing interventions is most appropriate in this situation? a) Observe the client for signs of petechiae and premature separation of the placenta b) Instruct the client to avoid wearing constrictive knee-high stockings c) Urge the client to discontinue the anticoagulant to prevent pregnancy complications d) Put the client on bed rest

a) Observe the client for signs of petechiae and premature separation of the placenta Rationale: Subclinical bleeding from continuous anticoagulant therapy in the woman has the potential to cause placental dislodgement. Observe a woman who is taking an anticoagulant for signs of petechiae and signs of premature separation of the placenta, therefore, during both pregnancy and labor. The nurse should not urge the client to discontinue the anticoagulant, as this is not within the nurse's scope of practice and, in any case, the client still needs the anticoagulant to prevent clots. Bed rest is prescribed for clients with a thrombus, to prevent it from moving and becoming a pulmonary embolus. Avoiding the use of constrictive knee-high stockings is to prevent thrombus formation.

A woman with an artificial mitral valve develops heart failure at the 20th week of pregnancy. Which of the following measures would you stress with her during the remainder of the pregnancy? a) Obtaining enough rest b) Beginning a low-impact aerobics program c) Discontinuing her prepregnancy anticoagulant d) Maintaining a high fluid intake

a) Obtaining enough rest Rationale: As the blood volume doubles during pregnancy, heart failure can occur. The pregnant woman needs to obtain adequate rest to prevent overworking the heart. Fluid may need to be restricted.

A young woman with scoliosis has just learned that she is pregnant. Several years ago, she had stainless-steel rods surgically implanted on both sides of her vertebrae to strengthen and straighten her spine. However, her pelvis is unaffected by the condition. Which of the following does the nurse anticipate in this woman's pregnancy? a) Potential for greater than usual back pain b) Cesarean birth c) Increased risk of fetal trauma d) Increased risk of miscarriage

a) Potential for greater than usual back pain Rationale: Surgical correction of scoliosis (lateral curvature of the spine) involves implanting stainless-steel rods on both sides of the vertebrae to strengthen and straighten the spine. Such rod implantations do not interfere with pregnancy; a woman may notice more than usual back pain, however, from increased tension on back muscles. If a woman's pelvis is distorted due to scoliosis, a cesarean birth may be scheduled to ensure a safe birth, but this is not required in this scenario. Vaginal birth, if permitted, requires the same management as for any woman. With the improved management of scoliosis, the high maternal and perinatal risks associated with the disorder reported in earlier literature no longer exist.

A 44-year-old client has lost several pregnancies over the last 10 years. For the past 3 months, she has had fatigue, nausea, and vomiting. She visits the clinic and takes a pregnancy test; the results are positive. Physical examination confirms a uterus enlarged to 13 weeks' gestation; fetal heart tones are heard. Ultrasound reveals that the client is experiencing some bleeding. Considering the client's prenatal history and age, what does the nurse recognize as the greatest risk for the client at this time? a) Pregnancy loss. b) Preterm labor. c) Premature birth. d) Hypertension.

a) Pregnancy loss. Rationale: The client's advanced maternal age (pregnancy in a woman 35 years or older) increases her risk for pregnancy loss. Hypertension, preterm labor, and prematurity are risks as this pregnancy continues. Her greatest risk at 13 weeks' gestation is losing this pregnancy.

A woman in week 35 of her pregnancy with severe hydramnios is admitted to the hospital. The nurse recognizes that which of the following is the biggest concern regarding this client? a) Preterm rupture of membranes followed by preterm birth b) Hemorrhaging c) Development of gestational trophoblastic disease d) Development of eclampsia

a) Preterm rupture of membranes followed by preterm birth Rationale: Even with precautions, in most instances of hydramnios, there will be preterm rupture of the membranes because of excessive pressure, followed by preterm birth. The other answers are not as big of concerns as preterm birth, in this situation.

A patient is admitted to labor and delivery for management of severe preeclampsia. An IV infusion of magnesium sulfate is started. What is the primary goal for magnesium sulfate therapy? a) Prevent maternal seizures b) Reverse edema c) Decrease blood pressure d) Decrease protein in urine

a) Prevent maternal seizures Rationale: The primary therapy goal for any preeclamptic patient is to prevent maternal seizures. Use of magnesium sulfate is the drug therapy of choice for severe preeclampsia and is only used to manage and attempt to prevent progression to eclampsia. Magnesium sulfate therapy does not have as its primary goal a decrease in blood pressure, a decrease in protein in the urine, nor the reversal of edema.

The following hourly assessments are obtained by the nurse on a patient with preeclampsia receiving Magnesium Sulfate: 97.3, P88, R10, blood pressure 148/110. What other priority physical assessment by the nurse should be implemented to assess for potential toxicity? a) Reflexes b) Magnesium sulfate level c) Lung sounds d) Oxygen saturation

a) Reflexes Rationale: Reflex assessment is part of the standard assessment for patients on magnesium sulfate. The first change when developing magnesium toxicity may be a decrease in reflex activity. The health care provider needs to be notified immediately. A change in lung sounds and oxygen saturation are not indicative of magnesium sulfate toxicity. Hourly blood draws to gain information on the magnesium sulfate level are not indicated.

When giving a postpartum client self-care instructions in preparation for discharge, the nurse instructs her to report heavy or excessive bleeding. How should the nurse describe "heavy bleeding?" a) Saturating 1 pad in 1 hour b) Saturating 1 pad in 6 hours c) Saturating 1 pad in 8 hours d) Saturating 1 pad in 3 hours

a) Saturating 1 pad in 1 hour Rationale: Bleeding is considered heavy when a woman saturates a sanitary pad in 1 hour. Excessive bleeding occurs when a postpartum client saturates 1 pad in 15 minutes. Moderate bleeding occurs when the bleeding saturates less than 15 cm of a pad in 1 hour.

A woman in labor is at risk for abruptio placentae. Which of the following assessments would most likely lead you to suspect that this has happened? a) Sharp fundal pain and discomfort between contractions. b) Painless vaginal bleeding and a fall in blood pressure. c) An increased blood pressure and oliguria. d) Pain in a lower quadrant and increased pulse rate.

a) Sharp fundal pain and discomfort between contractions. Rationale: An abruptio placentae refers to premature separation of the placenta from the uterus. As the placenta loosens, it causes sharp pain. Labor begins with a continuing nagging sensation. Painless vaginal bleeding and a fall in blood pressure are indicative of placenta previa. Pain in a lower quadrant and increased pulse rate are indicative of an ectopic pregnancy. Hypertension and oliguria are indicative of preeclampsia.

Sometimes an ectopic pregnancy occurs outside the woman's uterus. This usually occurs in one of the fallopian tubes. If the embryo continues to grow, it may rupture the tube. What are the signs and symptoms of a ruptured fallopian tube? a) Shoulder pain b) Unilateral abdominal pain c) Rectal pain d) Bilateral abdominal pain

a) Shoulder pain Rationale: Rarely, a woman may present with late signs, such as shoulder pain or hypovolemic shock. These signs are associated with tubal rupture, which occurs when the pregnancy expands beyond the tube's ability to stretch.

Which of the following would the nurse expect to find in a newborn of a mother who abuses heroin? a) Sneezing b) Easy consolability c) Vigorous sucking d) Hypotonicity

a) Sneezing Rationale: Newborns of mothers who abuse heroin or other narcotics display irritability, hypertonicity, a high-pitched cry, vomiting, diarrhea, respiratory distress, disturbed sleeping, sneezing, diaphoresis, fever, poor sucking, tremors, and seizures.

Working with pregnant teenagers as a special population requires the nurse to have knowledge of adolescent development. Which of the following is crucial for a positive pregnancy and outcome for the mother and fetus? a) Support network b) Cultural sensitivity c) Acceptance by peers d) Involvement of the father

a) Support network Rationale: One crucial part of management of teenage adolescent pregnancy includes helping the teens to develop an adequate support network. The network may include parents, teachers, friends and the father of the baby in addition to resources needed to provide care for the infant and self. Cultural sensitivity, involvement of the father, and acceptance by peers are important to the teenager who is pregnant, but they are not considered crucial for a positive pregnancy and outcome for the mother and fetus.

Review of a woman's labor and birth record reveals a laceration that extends through the anal sphincter muscle. The nurse identifies this as which of the following? a) Third-degree laceration b) Second-degree laceration c) Fourth-degree laceration d) First-degree laceration

a) Third-degree laceration Rationale: A third-degree laceration extends through the anal sphincter muscle. A first-degree laceration involves only skin and superficial structures above the muscle. A second-degree laceration extends through the perineal muscles. A fourth-degree laceration continues through the anterior rectal wall.

A pregnant woman in her second trimester comes to the prenatal clinic for a routine visit. She reports that she has a new kitten. The nurse would have the woman evaluated for which infection? a) Toxoplasmosis b) Parvovirus B19 c) Cytomegalovirus d) Herpes simplex virus

a) Toxoplasmosis Rationale: Toxoplasmosis is transferred by hand to mouth after touching cat feces while changing the litter box or through gardening in contaminated soil. Cytomegalovirus is transmitted via sexual contract, blood transfusions, kissing, and contact with children in daycare centers. Parvovirus B19 is a common self-limiting benign childhood virus that causes fifth disease. A pregnant woman may transmit the virus transplacentally to her fetus if she is exposed to an infected child. Herpesvirus infection occurs by direct contact of the skin or mucous membranes with an active lesion through kissing, sexual contact, or routine skin-to-skin contact.

As a rule, women can receive chemotherapy in the second and third trimesters without adverse fetal effects. a) True b) False

a) True Rationale: none

Postpartum infection is one event that is known to impede the recovery process of a new mother. Which characteristics after delivery make a woman more susceptible to infection? Select all that apply. a) Urinary stasis b) White blood cell count 25,000/mm³ c) Denuded endometrial arteries d) Episiotomy

a) Urinary stasis c) Denuded endometrial arteries d) Episiotomy Rationale: The urinary system after delivery is prone to infection, prompting a focus on cleanliness and frequent urination. The open uterine arteries are at risk for infection, as is any break in skin integrity. An elevated white blood cell count (from 10,000/mm³ to 30,000/mm³) is the body's defense against infection. A count greater than 30,000/mm³ or less than 10,000/mm³ prompts further investigation.

A nursing student is studying postpartal complications. Thromboembolic conditions have which of the following risk factors? (Select all that apply.) a) cigarette smoking b) anemia c) diabetes d) irritable bowel e) obesity f) multiparity

a) cigarette smoking b) anemia c) diabetes e) obesity f) multiparity Rationale: Risk factors for developing thromboembolic conditions include anemia, diabetes, cigarette smoking, obesity, preeclampsia, hypertension, varicose veins, pregnancy, cesarean section, multiparity, inactivity, and advanced maternal age.

Many patients experience a slight fever after delivery especially during the first 24 hours. To what should the nurse attribute this elevated temperature? a) dehydration b) change in the temperature from the delivery room c) infection d) fluid volume overload

a) dehydration Rationale: Many women experience a slight fever (100.4 degrees F) during the first 24 hours after delivery. This results from dehydration because of fluid loss during labor. With the replacement of fluids the temperature should return to normal after 24 hours.

When caring for a pregnant woman with cardiac problems, the nurse must be alert for signs and symptoms of cardiac decompensation (congestive heart failure), which include a) dyspnea, crackles, irregular weak pulse. b) regular heart rate, hypertension. c) shortness of breath, bradycardia, hypertension. d) increased urinary output, tachycardia, dry cough.

a) dyspnea, crackles, irregular weak pulse. Rationale: Signs of cardiac decompensation to congestive heart failure include crackles in the lungs from fluid, difficulty breathing, and weak pulse from heart exhaustion. The heart rate would not be regular, and a cough would not be dry. The heart rate would increase rather than decrease.

The nurse who works on a post-partum floor is mentoring a new graduate. She informs the new nurse that a post-partum assessment of the mother includes which of the following? (check all that apply) a) head-to-toe assessment b) vital signs of mother c) head-to-toe assessment of newborn d) newborn's vital signs e) pain level

a) head-to-toe assessment b) vital signs of mother e) pain level Rationale: Post-partum assessment of the mother usually includes vital signs, pain level and a systematic head-to-toe assessment of the mother. The others are care of the newborn and done by the nurse in the nursery.

Thirty minutes after receiving pain medication, a postpartum woman states that she sill has severe pain in the perineal region. Upon assessing and palpating the site, what can the nurse expect to find that might be causing this severe pain? a) hematoma b) infection c) nothing--it is normal d) DVT

a) hematoma Rationale: If a postpartum woman has severe perineal pain despite use of physical comfort measures and medication, the nurse should check for a hematoma by inspecting and palpating the area. If one is found, the nurse needs to notify the physician immediately.

A patient appears to be resting comfortably 12 hours after delivering her first child. In contrast, she labored for more than 24 hours, the physician had to use forceps to deliver the baby, and she had multiple vaginal examinations during labor. Based on this information what postpartum complication is the patient at risk for developing? a) infection b) depression c) hemorrhage d) pulmonary emboli

a) infection Rationale: There are many risk factors for developing a postpartum infection: operative procedures(eg, forceps, cesarean section, vacuum extraction), history of diabetes, prolonged labor (longer than 24 hours), use of Foley catheter, anemia, multiple vaginal examinations during labor, prolonged rupture of membranes, manual extraction of placenta, and HIV.

A nurse informs a pregnant woman with cardiac disease that she will need two rest periods each day and a full night's sleep. The nurse further instructs the patient that the best position for this rest is which of the following? a) left lateral recumbent b) right lateral recumbent c) prone d) on her back

a) left lateral recumbent Rationale: The pregnant woman should rest in the left lateral recumbent position to prevent supine hypotension syndrome and increased heart effort.

Hypercoagulability during pregnancy protects the mother against excessive blood loss during childbirth. It also can increase a woman's risk of developing a blood clot. It does this by which of the following ways? (Select all that apply.) a) localized vascular damage b) altered coagulation c) stasis d) decline in HGB e) decline in WBCs

a) localized vascular damage b) altered coagulation c) stasis Rationale: Three factors predispose women to thromboembolic disorders during pregnancy: stasis (compression of the large veins because of gravid uterus), altered coagulation (state of pregnancy), and localized vascular damage (may occur during birthing process). All these increase the risk of clot formation.

A nurse is auscultating the lungs of a postpartum patient and notices crackles and some dyspnea. The patient's respiratory rate is 12 breaths/min; she appears in some distress. What complication should the nurse suspect based on these data? a) pulmonary edema b) fluid volume deficit c) hemorrhage d) infection

a) pulmonary edema Rationale: Any change in the respiratory rate of a postpartum woman might indicate pulmonary edema, atelectasis, or pulmonary embolism and must be reported. Lungs should be clear upon auscultation.

A nurse is instructing students on how to check an episiotomy and perineum of a woman after Which of the following are normal in the early postpartum period? (Select all that apply.) a) slight bruising b) discharge c) redness d) edema

a) slight bruising d) edema Rationale: During the early postpartum period, the perineal tissue surrounding the episiotomy is typically edematous and slightly bruised. The normal episiotomy site should not have redness or discharge.

A woman of 16 weeks' gestation telephones you because she has passed some "berry-like" blood clots and now has continued dark brown vaginal bleeding. Which of the following would you instruct the woman to do? a) "Come to the health care facility if uterine contractions begin." b) "Come to the health facility with any vaginal material passed." c) "Maintain bed rest and count the number of perineal pads used." d) "Continue normal activity, but take your pulse every hour."

b) "Come to the health facility with any vaginal material passed." Rationale: This is a typical time in pregnancy for gestational trophoblastic disease to present. Asking the woman to bring any material passed vaginally would be important so it can be assessed for this.

A mother just delivered 3 hours ago. The nurse enters the room to continue hourly assessments and finds the patient on the phone telling the listener about her fear while driving to the hospital and not making it in time. The mother finishes the call, and the nurse begins her assessment with which phrase? a) "If you plan to breastfeed, you need to calm down." b) "It sounded like you had quite a time getting here. Would you like to continue your story?" c) "You have a beautiful baby, why worry about that now?" d) "I need to assess your fundus now."

b) "It sounded like you had quite a time getting here. Would you like to continue your story?" Rationale: The mother is going through the taking-in phase of relating events during her pregnancy and delivery. The nurse can facilitate this phase by allowing the mother to express herself. Diverting the conversation, admonishing the mother, or warning of potential problems does not accomplish this facilitation.

A client who gave birth vaginally 16 hours ago states she doesn't need to void at this time. The nurse reviews the documentation and finds that the client hasn't voided for 7 hours. Which response by the nurse is indicated? a) "If you don't attempt to void, I'll need to catheterize you." b) "It's not uncommon after delivery for you to have a full bladder even though you can't sense the fullness." c) "I'll check on you in a few hours." d) "I'll contact your physician."

b) "It's not uncommon after delivery for you to have a full bladder even though you can't sense the fullness." Rationale: After a vaginal delivery, the client should be encouraged to void every 4 to 6 hours. As a result of anesthesia and trauma, the client may be unable to sense the filling bladder. It is premature to catheterize the client without allowing her to attempt to void first. There is no need to contact the physician at this time, because the client is demonstrating common adaptations in the early postpartum period. Allowing the client's bladder to fill for another 2 to 3 hours might cause overdistention.

A woman's baby is HIV positive at birth. She asks the nurse if this means the baby will develop AIDS. Which of the following statements would be the nurse's best answer? a) "She already has AIDS. That's what being HIV positive means." b) "The antibodies may be those transferred across the placenta; the baby may not develop AIDS." c) "HIV is transmitted at birth; having a cesarean birth prevented transmission." d) "HIV antibodies do not cross the placenta; this means the baby will develop AIDS."

b) "The antibodies may be those transferred across the placenta; the baby may not develop AIDS." Rationale: Infants born of HIV-positive women test positive for HIV antibodies at birth because these have crossed the placenta. An accurate disease status cannot be determined until the antibodies fade at about 18 months. Testing positive for HIV antibodies does not mean the infant has AIDS. Having a cesarean birth does decrease the risk of transmitting the virus to the infant at birth; it does not prevent the transmission of the disease. HIV antibodies do cross the placenta, which is why babies born of HIV positive mothers are HIV positive.

A client is 11 weeks pregnant after many years trying to conceive. After arriving home from a normal prenatal visit, she experiences mild cramping and has a gush of bright red vaginal bleeding. She calls the nurse and reports having soaked a pad with fresh blood in fewer than 30 minutes. The uterine cramping is worsening. What is the most appropriate response from the nurse? a) "This is nothing to worry about. Many women bleed during pregnancy." b) "You need to seek immediate attention from the primary care provider." c) "I am sorry. There is nothing you can do, because you are likely miscarrying." d) "Lie down and call your health care provider tomorrow if symptoms continue."

b) "You need to seek immediate attention from the primary care provider." Rationale: Pregnancy loss during the early weeks of pregnancy may seem like a heavy menstrual period. A primary care provider should assess blood loss of this amount with or without uterine cramping as soon as possible.

A pregnant woman with diabetes is having a glycosylated hemoglobin level drawn. Which result would require the nurse to revise the client's plan of care? a) 6.0% b) 8.5% c) 7% d) 5.5%

b) 8.5% Rationale: A glycosylated hemoglobin level of more than 8% indicates poor control and the need for intervention, necessitating a revision in the woman's plan of care.

A client in her first trimester has just experienced a miscarriage. The nurse knows that which of the following is the most likely cause of the miscarriage? a) Lack of sufficient progesterone produced by the corpus luteum b) Abnormal fetal development c) Rejection of the embryo through an immune response d) Implantation abnormality

b) Abnormal fetal development Rationale: The most frequent cause of miscarriage in the first trimester of pregnancy is abnormal fetal development, due either to a teratogenic factor or to a chromosomal aberration. In other miscarriages, immunologic factors may be present or rejection of the embryo through an immune response may occur. Another common cause of early miscarriage involves implantation abnormalities. Miscarriage may also occur if the corpus luteum on the ovary fails to produce enough progesterone to maintain the decidua basalis.

A woman with a long history of controlled asthma has just had her first antenatal visit for her fourth child. She is late for a meeting and says she knows what to do. What is the best action the nurse can take? a) Note in the chart that the woman was not counseled about her asthma. b) Acknowledge her need to leave but ask her to demonstrate the use of her inhaler and her peak flow meter before she goes; make any necessary corrections to her technique. Remind her to take her regular medications. c) Remind her to continue taking her asthma medications, to monitor her peak flow daily, and to monitor the baby's kicks in the second and third trimesters. d) Schedule an appointment for her to return to discuss her asthma management.

b) Acknowledge her need to leave but ask her to demonstrate the use of her inhaler and her peak flow meter before she goes; make any necessary corrections to her technique. Remind her to take her regular medications. Rationale: Management of asthma during pregnancy is very important, the nurse must document the patient has the proper ability to manage her asthma for her health and the health of the fetus. Reminding the patient to continue taking her prescribed medication to monitor her peak flow daily is not enough. It is the nurse's responsibility to KNOW that the patient knows how to take her medications. Monitoring the baby's kicks in the second and third trimester is an appropriate action. Scheduling a return appointment to discuss asthma management is not appropriate. She could have an asthma attack between the time you see her and the time you schedule a return appointment. Noting in the chart that the woman was not counseled does not relieve the nurse of her obligation to ensure that the woman knows how to use her inhaler and her peak flow meter.

A postpartum mother has the following lab data recorded: RH negative and rubella titer is positive. What is the appropriate nursing intervention? a) Assess the rubella of the baby b) Administer RhoGam within 72 hours. c) Administer rubella vaccine before discharge. d) Assess the RH of the baby.

b) Administer RhoGam within 72 hours. Rationale: The rubella is a virus and the mother has a positive titer indicating she is immune, this is important data for prenatal care and only has a bearing if the patient were negative. The RH is negative for the mother and the infant status is unknown, to protect future pregnancies the mother should be given RhoGam. It would not be appropriate to administer the rubella vaccine, assess the rubella titer of the baby, or assess the Rh status of the baby.

Which of the following findings would lead you to suspect that a woman is developing a postpartum complication? a) Lochia appearing pinkish-brown on the fourth day b) An absence of lochia c) Red-colored lochia for the first 24 hours d) Lochia that is the color of menstrual blood

b) An absence of lochia Rationale: Women should have a lochia flow following childbirth. Absence of a flow is abnormal; it suggests dehydration from infection and fever.

Seven hours ago, a G5 P4014 woman delivered a 4133-g male infant. She has voided once and calls for a nurse to check because she states that she feels "really wet" now. Upon examination, her perineal pad is saturated. The immediate nursing action is to a) Increase the flow of an IV b) Assess and massage the fundus c) Inspect the perineum for lacerations d) Call the physician or the nurse-midwife

b) Assess and massage the fundus Rationale: This woman is a multigravida who delivered a large baby and is at risk for hemorrhage. The other actions are to be done after the initial fundal massage.

A woman has been diagnosed as having pregnancy-induced hypertension. Which of the following is the most typical symptom of this? a) Increased perspiration b) Blood pressure elevation c) Weight loss d) Susceptibility to infection

b) Blood pressure elevation Rationale: The symptom of hypertension of pregnancy is blood pressure elevation (140/90 mm Hg).

When providing counseling on early pregnancy loss, the nurse should include what as the most common cause for spontaneous abortion? a) Maternal smoking b) Chromosomal defect c) Lack of prenatal care d) The age of the mother

b) Chromosomal defect Rationale: The most common cause for the loss of a fetus in the first trimester is associated with a genetic defect or chromosomal defect. There is nothing that can be done and the mother should feel no fault. The nurse needs to educate the parents to speak with a health care provider for further information and questions related to genetic testing. Early pregnancy loss is not associated with maternal smoking, lack of prenatal care, or the age of the mother.

A pregnant client with a history of heart disease has been admitted to a healthcare center with complaints of breathlessness. The client also complains of shortness of breath and easy fatigue when doing ordinary activity. The client's condition is markedly compromised. The nurse would document the client's condition using the New York Heart Association (NYHA) classification system as which class? a) Class IV b) Class III c) Class I d) Class II

b) Class III Rationale: The nurse should classify the client's condition as belonging to class III of NYHA. In class III of NYHA classification, the client will be symptomatic with ordinary activity and her condition is markedly compromised. The client is asymptomatic with all kinds of activity and is in uncompromised state in class I. The client is symptomatic with increased activity and is in slight compromised state in class II. The client is symptomatic when resting and is incapacitated in class IV.

A pregnant client with multiple gestation arrives at the maternity clinic for a regular antenatal check up. The nurse would be aware of the client's risk of perinatal complications including which of the following? a) Maternal hypotension b) Congenital anomalies c) Post-term birth d) Fetal non-immune hydrops

b) Congenital anomalies Rationale: Multiple gestation involves two or more fetuses. The perinatal complications associated with multiple pregnancy include preterm birth, maternal hypertension and congenital anomalies. Fetal non-immune hydrops occurs in the infection of pregnant clients with parvovirus. Post-term birth, maternal hypotension and fetal non-immune hydrops are not seen as complications of multiple pregnancy.

A woman gave birth vaginally approximately 12 hours ago and her temperature is now 100 degrees F. Which action would be most appropriate? a) Notify the health care provider about this elevation; this finding reflects infection. b) Continue to monitor the woman's temperature every 4 hours; this finding is normal. c) Obtain a urine culture; the woman most likely has a urinary tract infection. d) Inspect the perineum for hematoma formation.

b) Continue to monitor the woman's temperature every 4 hours; this finding is normal. Rationale: A temperature of 100.4 degrees F or less during the first 24 hours postpartum is normal and may be the result of dehydration due to fluid loss during labor. There is no need to notify the physician, obtain a urine culture, or inspect the perineum (other than the routine assessment of the perineum), because this finding is normal.

Which of the following nursing diagnoses would be most appropriate for a woman diagnosed with pregnancy-induced hypertension? a) Risk for injury related to fetal distress b) Deficient fluid volume related to vasospasm of arteries c) Imbalanced nutrition related to decreased sodium levels d) Ineffective tissue perfusion related to poor heart contraction

b) Deficient fluid volume related to vasospasm of arteries Rationale: Hypertension of pregnancy is caused by vascular spasm. This leads to increased blood pressure and edema. Extensive edema leads to a deficiency of fluid volume.

You encourage a woman with gestational diabetes to maintain an active exercise period during pregnancy. Prior to this exercise period, you would advise her to do which of the following? a) Eat a high-carbohydrate snack b) Eat a sustaining-carbohydrate snack c) Inject a bolus of insulin d) Add a bolus of long-acting insulin

b) Eat a sustaining-carbohydrate snack Rationale: Because exercise uses up glucose, women with diabetes should take a sustaining-carbohydrate snack before hard exercise to prevent hypoglycemia.

A client visits a health care facility with complaints of amenorrhea for 10 weeks, fatigue, and breast tenderness. Which of the following additional signs and symptoms suggest the presence of molar pregnancy? Select all that apply. a) Whitish discharge from the vagina b) Elevated hCG levels c) Hyperemesis gravidarum d) Dyspareunia e) Absence of fetal heart sound

b) Elevated hCG levels c) Hyperemesis gravidarum e) Absence of fetal heart sound Rationale: The signs and symptoms of molar pregnancy include an elevated hCG level, absence of fetal heart sounds, and hyperemesis gravidarum. Whitish discharge from the vagina and dyspareunia (painful sexual intercourse) are seen in cases of infection, and not in molar pregnancy. In molar pregnancy a brownish vaginal bleeding is seen.

Which of the following is an appropriate nursing intervention for prevention of a urinary tract infection (UTI) in the postpartum woman? a) Increasing intravenous fluids. b) Encouraging the woman to empty her bladder completely every 2 to 4 hours. c) Increasing oral fluid intake. d) Screening for bacteriuria in the urine.

b) Encouraging the woman to empty her bladder completely every 2 to 4 hours. Rationale: The nurse should advise the woman to urinate every 2 to 4 hours while awake to prevent overdistention and trauma to the bladder. Maintaining a good fluid intake is also important, but it is not necessary to increase fluids if the woman is consuming enough. Screening for bacteria in the urine would require a physician's order and is not necessary as a prevention measure.

A novice nurse asks to be assigned to the least complex antepartum patient. Which of the following conditions would necessitate the least complex care requirements? a) Pre-ecalmpsia. b) Gestational hypertension. c) Placenta previa. d) Abruptio placenta.

b) Gestational hypertension. Rationale: Hypertensive disorders represent the most common complication of pregnancy. Gestational hypertension is elevated blood pressure without proteinuria, other signs of pre-eclampsia, or pre-existing hypertension. Abruptio placenta (separation of the placenta from the uterine wall), placenta previa (placenta covering the cervical os), and pre-eclampsia are high-risk, potentially life-threatening conditions for the fetus and mother during labor and birth.

To which of the following patients being discharged for home must the nurse stress that it is absolutely critical the patient return for monthly follow-up visits? A woman who: a) Has experienced a complete spontaneous abortion b) Has experienced a molar pregnancy c) Has experienced an ectopic pregnancy d) Is Rh negative

b) Has experienced a molar pregnancy Rationale: Molar pregnancies can indicate the possibility of developing malignancy. The woman will need close observation and follow-up for the year following the diagnosis. Follow-up visits after an ectopic pregnancy or a complete spontaneous abortion are typically scheduled at six weeks, not monthly. A woman who is Rh negative does not need a follow-up visit because of her Rh status.

When educating a pregestational patient on how to control her blood sugar, the nurse knows there are three main facets to glycemic control: diet, exercise and _______. Which of the following is the third facet? a) Glucose tablets b) Insulin c) Folic acid d) Niacin

b) Insulin Rationale: The three main facets to glycemic control for the woman with pregestational diabetes are diet, exercise, and insulin. Folic acid does not impact glycemic control. Glucose tablets are not a facet of glycemic control.

Inspection of a woman's perineal pad reveals a 5-inch stain. The nurse documents this amount as which of the following? a) Scant b) Moderate c) Heavy d) Light

b) Moderate Rationale: Moderate lochia would describe a 4- to 6-inch stain, scant lochia a 1- to 2-inch stain, and light or small an approximately 4-inch stain. Heavy or large lochia would describe a pad that is saturated within 1 hour.

A pregnant client with severe pre-eclampsia has developed the HELLP syndrome. In addition to the observations necessary for pre-eclampsia, what other nursing intervention is critical for this patient? a) Administration of a tocolytic, if prescribed b) Observation for bleeding c) Monitoring for infection d) Maintaining a patent airway

b) Observation for bleeding Rationale: Because of the low platelet count associated with this condition, women with the HELLP syndrome need extremely close observation for bleeding, in addition to the observations necessary for pre-eclampsia. Maintaining a patent airway is a critical intervention needed for a patient with eclampsia while she is having a seizure. Administration of a tocolytic would be appropriate for halting labor. Monitoring for infection is not a priority intervention in this situation.

The nurse explains to a pregnant patient that she will need to take iron during her pregnancy after being diagnosed with iron-deficiency anemia. The nurse suggests that absorption of the supplemental iron can be increased by taking it with which of the following? a) Milk b) Orange juice c) Meals high in iron d) Legumes

b) Orange juice Rationale: Anemia is a condition in which the blood is deficient in red blood cells, from an underlying cause. The woman needs to take iron to manufacture enough red blood cells. Taking an iron supplement will help improve her iron levels, and taking iron with foods containing ascorbic acid, such as orange juice, improves the absorption of iron.

A pregnant woman with preeclampsia is to receive magnesium sulfate IV. Which of the following assessments would be most important prior to administering a new dose? a) Anxiety level b) Patellar reflex c) Blood pressure d) Pulse rate

b) Patellar reflex Rationale: A symptom of magnesium sulfate toxicity is loss of deep tendon reflexes. Assessing for one of these before administration is assurance the drug administration will be safe.

The nurse is teaching a pregnant woman with iron deficiency anemia about foods high in iron. Which food(s) if selected by the woman indicates a successful teaching program? Select all that apply. a) Potatoes b) Peanut butter c) Corn d) Raisins e) Yogurt f) Broccoli

b) Peanut butter d) Raisins f) Broccoli Rationale: Foods high in iron include dried fruits such as raisins, whole grains, green leafy vegetables such as broccoli and spinach, peanut butter, and iron-fortified cereals. Potatoes and corn are high in carbohydrates. Yogurt is a good source of calcium.

A client in her fifth month of pregnancy is having a routine clinic visit. The nurse should assess the client for which common second trimester condition? a) Metabolic alkalosis b) Physiological anemia c) Mastitis d) Respiratory acidosis

b) Physiological anemia Rationale: Hemoglobin level and hematocrit decrease during pregnancy as the increase in plasma volume exceeds the increase in red blood cell production. Mastitis is an infection in the breast characterized by a swollen tender breast and flulike symptoms. This condition is most commonly seen in breast-feeding clients. Alterations in acid-base balance during pregnancy result in a state of respiratory alkalosis, compensated by mild metabolic acidosis.

When palpating for fundal height on a postpartal woman, which technique is preferable? a) Resting both hands on the fundus b) Placing one hand at the base of the uterus, one on the fundus c) Palpating the fundus with only fingertip pressure d) Placing one hand on the fundus, one on the perineum

b) Placing one hand at the base of the uterus, one on the fundus Rationale: Supporting the base of the uterus before palpation prevents the possibility of uterine inversion with palpation.

A woman in week 35 of her pregnancy with severe hydramnios is admitted to the hospital. The nurse recognizes that which of the following is the biggest concern regarding this client? a) Development of gestational trophoblastic disease b) Preterm rupture of membranes followed by preterm birth c) Development of eclampsia d) Hemorrhaging

b) Preterm rupture of membranes followed by preterm birth Rationale: Even with precautions, in most instances of hydramnios, there will be preterm rupture of the membranes because of excessive pressure, followed by preterm birth. The other answers are not as big of concerns as preterm birth, in this situation.

A client in her 38th week of gestation is admitted into the labor and birth unit with painless bleeding from the vagina. The client is diagnosed with placenta previa. When reviewing the client's history, which of the following would the nurse identify as a risk factor for placenta previa? a) Maternal age more than 30 years b) Previous cesarean birth c) Primigravida status d) Living in coastal areas

b) Previous cesarean birth Rationale: A previous cesarean birth is a risk factor for developing placenta previa. This is due to the damage caused to the endometrial tissue. Multiparity, and not a primigravida status, predisposes to placenta previa. Maternal age over 35 years, and not just more than 30 years, is considered another risk factor. Placenta previa is more common among those living in high altitudes, and not among those living in coastal areas.

What makes the diagnosis of gestational hypertension different from the diagnosis of preeclampsia? a) Ketonuria b) Proteinuria c) Severity of hypertension d) The hypertension of gestation disappears after delivery. The hypertension of preeclampsia does not.

b) Proteinuria Rationale: Gestational hypertension is the current term used to describe elevated blood pressure (greater than or equal to 140/90 mm/Hg) that develops for the first time during pregnancy without the presence of protein in the urine.

A nurse is explaining to a group of nursing students that eclampsia or seizures in pregnant women are preceded by an acute increase in maternal blood pressure. Which of the following are features of an acute increase in blood pressure? Select all that apply. a) Auditory hallucinations b) Proteinuria c) Hypereflexia d) Blurring of vision e) Hyperglycemia

b) Proteinuria c) Hypereflexia d) Blurring of vision Rationale: Eclampsia is usually preceded by an acute increase in blood pressure as well as worsening signs of multi-organ system failure seen as increasing liver enzymes, proteinuria, and symptoms such as blurred vision and hyperreflexia. Hyperglycemia and auditory hallucinations are not seen with an acute increase in maternal BP.

A patient with preeclampsia is receiving magnesium sulfate. Which of the following nursing assessments should be ongoing while the medication is being administered? a) Hemoglobin. b) Respiratory rate. c) Urine protein. d) Ability to sleep.

b) Respiratory rate. Rationale: The level of magnesium in therapeutic range is 4 to 8 mg/dL. If magnesium toxicity occurs, one sign in the patient will be a decrease in the respiratory rate and a potential respiratory arrest. Respiratory rate will be monitored when on this medication. The patient's hemoglobin and ability to sleep are not factors for on-going assessments for the patient on magnesium sulfate. Urinary output is measured hourly on the preeclamptic patient receiving magnesium sulfate, but urine protein is not an ongoing assessment.

A client is diagnosed with peripartum cardiomyopathy (PPCM). Which of the following would the nurse expect to administer to the client? a) Methadone therapy b) Restricted sodium intake c) Ginger therapy d) Monoamine oxidase inhibitors

b) Restricted sodium intake Rationale: The client with peripartum cardiomyopathy should be prescribed restricted sodium intake to control the BP. Monoamine oxidase inhibitors are given to treat depression in pregnancy, and not peripartum cardiomyopathy. Methadone is a drug given for the treatment of substance abuse during pregnancy. Complimentary therapies like ginger therapy help in the alleviation of hyperemesis gravidarum, and not peripartum cardiomyopathy.

When dealing with a pregnant adolescent, the nurse assists the client to integrate the tasks of pregnancy while at the same time fostering development of which of the following? a) Trust b) Self-identity c) Autonomy d) Dependence

b) Self-identity Rationale: The nurse assists the pregnant adolescent to integrate the tasks of pregnancy, bonding, and preparing to care for another with the tasks of developing self-identity and independence. Trust is a developmental task of infancy. Autonomy is a developmental task of toddlerhood. Independence, not dependence, is fostered.

A pregnant woman at 12 weeks' gestation calls you because she has begun minimal fresh vaginal spotting. She is distressed because her physician says she is not going to do anything for her but "wait and see." Which of the following would you suggest? a) Suggest she take an over-the-counter tocolytic just to feel secure. b) Tell her that medication to prolong a 12-week pregnancy usually is not advised. c) Advise her to ask for a second physician opinion. d) Explain that her doctor meant for her to maintain strict bed rest by "wait and see."

b) Tell her that medication to prolong a 12-week pregnancy usually is not advised. Rationale: Because many early pregnancy losses occur as the result of chromosome abnormalities, an aggressive approach to prolong these is not usually recommended.

A pregnant woman at 12 weeks' gestation calls you because she has begun minimal fresh vaginal spotting. She is distressed because her physician says she is not going to do anything for her but "wait and see." Which of the following would you suggest? a) Suggest she take an over-the-counter tocolytic just to feel secure. b) Tell her that medication to prolong a 12-week pregnancy usually is not advised. c) Explain that her doctor meant for her to maintain strict bed rest by "wait and see." d) Advise her to ask for a second physician opinion.

b) Tell her that medication to prolong a 12-week pregnancy usually is not advised. Rationale: Because many early pregnancy losses occur as the result of chromosome abnormalities, an aggressive approach to prolong these is not usually recommended.

An episiotomy or a cesarean incision requires assessment. Which assessment criterion for skin integrity is not initially noted? a) Edema b) Temperature c) Drainage d) Redness

b) Temperature Rationale: The temperature of an incision would be determined only if the other parameters require this. A sterile glove would be used to assess skin temperature.

A woman and her partner present at her first antenatal obstetrics appointment. She is 6 weeks pregnant. Her blood tests show that she is Rh negative although she has not previously known this. She has no other children but suspects that she miscarried early in a pregnancy 2 years previously. She felt fine afterward so she never received medical attention for that suspected episode. Her partner reports that he is also Rh negative. What action will the nurse be likely to take? a) If an antibody screen is positive, the nurse will administer RhoGam to the patient. b) The nurse will perform all normal procedures and follow-up tasks because the Rh status of the patient and the child are not a concern at this time. c) The nurse will arrange for an amniocentesis to assess hemolytic disease in the fetus. d) If an antibody screen is negative, the nurse will administer RhoGam to the patient.

b) The nurse will perform all normal procedures and follow-up tasks because the Rh status of the patient and the child are not a concern at this time. Rationale: If the woman is Rh negative and her partner is Rh negative the fetus will also be Rh negative and the woman will not require treatment with RhoGam. Therefore options A, B, and C are incorrect.

A young woman presents at the emergency department with complaints of lower abdominal cramping and spotting at 12 weeks' gestation. The physician performs a pelvic examination and finds that the cervix is closed. What does the physician suspect is the cause of the cramps and spotting? a) Cervical insufficiency b) Threatened abortion c) Habitual abortion d) Ectopic pregnancy

b) Threatened abortion Rationale: Spontaneous abortion occurs along a continuum: threatened, inevitable, incomplete, complete, missed. The definition of each category is related to whether or not the uterus is emptied, or for how long the products of conception are retained.

You are doing a nursing assessment on a new patient in the obstetric clinic. The woman estimates that she is approximately 16 weeks pregnant. While assessing her you ask about what appear to be scratch marks on her hands, and she tells you that she has three cats at home. What screening would be ordered for this woman? a) Cytomegalovirus b) Toxoplasmosis c) Herpes Simplex Virus d) Hepatitis C

b) Toxoplasmosis Rationale: Toxoplasmosis is an infection caused by the protozoan Toxoplasma gondii, also referred to as T. gondii. Transmission is via undercooked meat and through cat feces. Toxoplasmosis is a common infection in humans and usually produces no symptoms. However, when the infection passes from the woman through the placenta to the fetus, a condition called congenital toxoplasmosis can occur. Approximately 400 to 4,000 cases of congenital toxoplasmosis occur per year in the United States (Williams, 2007). The classic triad of symptoms for congenital toxoplasmosis is chorioretinitis, intracranial calcification, and hydrocephalus in the newborn.

A pregnant single mom living alone tells the nurse she is considering getting a cat for her two year old daughter. Which is the best response by the nurse? a) The exposure to the cat litter may cause you to need a C-section b) You should wait until after you deliver to obtain the cat for your daughter c) This will cut down on the jealousy for your two year old when the baby comes d) If you don't think caring for a cat is too much work, that would be great

b) You should wait until after you deliver to obtain the cat for your daughter Rationale: Toxoplasma gondii is a protozoan that can be transmitted via undercooked meat and through cat litter. Having a cat is not an issue, but cleaning the litter box may expose the mother to the infection and result in fetal anomalies. Option A is incorrect; exposure to the cat litter will not necessitate a cesarean section. Option B is incorrect; having a cat will not cut down on any jealousy the 2-year-old might feel when the new baby is born. Option C is incorrect; the nurse would not encourage the mother to get her child a cat until after the new baby is born.

A nurse finds the uterus of a postpartum woman to be boggy and somewhat relaxed. This a sign of which of the following? a) normal involution b) atony c) hemorrhage d) infection

b) atony Rationale: The uterus in a postpartum patient should be midline and firm. A boggy or relaxed uterus signifies uterine atony, which can predispose the woman to hemorrhage.

A new mother tells the nurse at the baby's 3 month check-up, "When she cries, it seems like I am the only one who can calm her down." This is an example of which of the following? a) bonding b) attachment c) being spoiled d) none of the above

b) attachment Rationale: Attachment is the development of strong affection between an infant and a significant other. It does not occur overnight. It occurs through mutually satisfying experiences. Attachment behaviors include seeking, staying close to, and exchanging gratifying experiences with the infant. Bonding is the close emotional attraction to a newborn by the parents that develops in the first 30 to 60 minutes after birth. This is not an example of being spoiled.

A nurse is instructing a patient who is breastfeeding for the first time that before her milk comes in she should expect to see colostrum, which is best described as which of the following? a) milky white b) creamy yellow c) gray liquid d) bluish white

b) creamy yellow Rationale: If a woman has any discharge from her nipples postpartum, it should be described and documented if it is not colostrum (creamy yellow) or foremilk (bluish white).

The nurse working on a postpartum must check lochia in terms of amount, color, change with activity and time, and: a) pH b) odor c) specific gravity d) consistency

b) odor Rationale: The nurse when assessing lochia must do so in terms of amount, color, odor, and change with activity and time.

A woman you care for has an Rh-negative blood type. Following the birth of her infant, you administer her RhIG (D immune globulin). The purpose of this is to a) promote maternal D antibody formation. b) prevent maternal D antibody formation. c) stimulate maternal D immune antigens. d) prevent fetal Rh blood formation.

b) prevent maternal D antibody formation. Rationale: Because RhIG contains passive antibodies, the solution will prevent the woman from forming long-lasting antibodies.

A nursing student learns that a certain condition in 1 in every 2,000 pregnancies is a major cause of death. What is this condition? a) infection b) pulmonary embolism c) hemorrhage d) hypertension

b) pulmonary embolism Rationale: Pulmonary embolism occurs in 1 in 2,000 pregnancies and is a major cause of maternal mortality.

When an infant smiles at the mother and the mother in turn smiles and kisses her baby, this would be which phase of attachment? a) proximity b) reciprocity c) commitment d) all of the above

b) reciprocity Rationale: Proximity refers to the physical and psychological experience of the parents being close to their infant. Reciprocity is the process by which the infant's abilities and behaviors elicit parental responses (ie, the smile by the infant gets a smile and kiss in return). Commitment refers to the enduring nature of the relationship.

After teaching a postpartum woman about postpartum blues, which statement indicates effective teaching? a) "I'll need to take medication to treat the anxiety and sadness." b) "If the symptoms last more than a few days, I need to call my doctor." c) "I might feel like laughing one minute and crying the next." d) "I should call this support line only if I hear voices."

c) "I might feel like laughing one minute and crying the next." Rationale: Emotional lability is typical of postpartum blues. Further evaluation is necessary if symptoms persist for more than 2 weeks. Postpartum blues are usually self-limiting and require no medication. Support lines can be used whenever the woman feels down.

Select the statement by the pregnant woman that indicates the need for more teaching about preeclampsia. a) "I will count my baby's movements twice a day." b) "If I have a slight headache I'll take Tylenol and call if unrelieved." c) "If I have changes in my vision, I will lie down and rest." d) "I will weigh myself every morning after voiding before breakfast."

c) "If I have changes in my vision, I will lie down and rest." Rationale: Changes in the visual field may indicate the patient has moved from preeclampsia to severe preeclampisa and is at risk for developing a seizure due to changes in cerebral blood flow. The patient would require immediate assessment and intervention. Options B, C, and D would not indicate that more teaching about preeclampsia is indicated.

A pregnant woman who has had cardiovascular disease for the last 3 years asks the nurse why this disorder makes her pregnancy an "at-risk" pregnancy. What is the nurse's best response? a) "The fact that you are receiving prenatal care will help." b) "Our facility has a lot of experience in dealing with this." c) "Pregnancy taxes the circulatory system of every woman." d) "Don't worry. You have an excellent doctor."

c) "Pregnancy taxes the circulatory system of every woman." Rationale: Pregnancy taxes the circulatory system of every woman because both the blood volume and cardiac output increase by approximately 30% to 50%. Half of these increases occur by 8 weeks; they are maximized by mid-pregnancy.

A nursing instructor teaching students how to check the patient's uterus postpartum realizes that further instruction is needed when one of the students says: a) "Normally the fundus progresses downward at a rate of 1 fingerbreadth per day after birth." b) "One to two hours after birth the fundus is typically between the umbilicus and symphysis pubis." c) "Six to twelve hours after birth the fundus is typically at the level of the umbilicus." d) "One to two hours after birth the fundus is typically at the level of the umbilicus."

c) "Six to twelve hours after birth the fundus is typically at the level of the umbilicus." Rationale: One to two hours after birth the fundus is typically between the umbilicus and symphysis pubis. At 6ix to 12 hours after birth the fundus usually is at the level of the umbilicus. Normally the fundus progresses downward at at rate of one fingerbreadth per day after birth.

A client is 33 weeks pregnant and has had diabetes since age 21. When checking her fasting blood glucose level, which value would indicate the client's disease is controlled? a) 45 mg/dl b) 120 mg/dl c) 85 mg/dl d) 136 mg/dl

c) 85 mg/dl Rationale: Recommended fasting blood glucose levels in pregnant clients with diabetes are 60 to 95 mg/dl. A fasting blood glucose level of 45 g/dl is low and may result in symptoms of hypoglycemia. A blood glucose level below 120 mg/dl is recommended for 2-hour postprandial values. A blood glucose level above 136 mg/dl in a pregnant client indicates hyperglycemia.

A woman who is 31 weeks pregnant presents at the emergency room with bright red vaginal bleeding. She says the onset of the bleeding was sudden and she has no pain. The nurse is most likely to assist the physician or technician with which exam? a) A blood transfusion b) An abdominal ultrasound c) A transvaginal ultrasound d) A digital cervical exam

c) A transvaginal ultrasound Rationale: The use of a transvaginal ultrasound is the diagnostic test of choice; it is 100% accurate in prediction of placenta previa, while abdominal ultrasound is only 95% accurate. A digital cervical exam is contraindicated in this patient and the scenario described does not indicate the need for a blood transfusion.

The nurse is caring for a woman who is 28 weeks pregnant and has been hospitalized with moderate bleeding which seems to be stabilizing; she is asleep on her side. While checking the fetal monitor tape the nurse notices that the baseline has gradually increased with late decelerations. What is the first action the nurse will take? a) Reposition the mother in a semirecumbent position on her back. b) Notify the physician. c) Administer oxygen to the mother. d) Decrease the rate of IV fluids.

c) Administer oxygen to the mother. Rationale: The fetus is showing signs of fetal distress. The immediate treatment is using oxygen, putting the patient in a side lying position, and increasing IV fluids. The interventions are to improve fetal circulation. Therefore options B, C, and D are incorrect answers.

A postpartum mother has the following lab data recorded: a negative rubella titer. What is the appropriate nursing intervention? a) Assess the rubella of the baby b) No action needed. c) Administer rubella vaccine before discharge. d) Notify the health care provider.

c) Administer rubella vaccine before discharge. Rationale: Rubella is a virus, which when contracted during pregnancy has significant complications for the fetus. The illness is mild to the adult but can result in the infant being born deaf and blind. There is no cure, the CDC recommends all individuals be vaccinated against rubella. If the titer is negative, the mother does not have protection against rubella and the next pregnancy would be at risk. She should receive the vaccination prior to discharge from the hospital. This makes option A incorrect. Assessing the rubella titer of the baby would not mean anything. The baby has not had rubella and has not received antibodies against rubella from the mother. Notifying the health care provider is not a priority, as most institutions have standing orders to administer the rubella vaccine if the mother's rubella titer is negative.

A pregnant client is screened for tuberculosis during her first prenatal visit. An intradermal injection of purified protein derivative (PPD) of the tuberculin bacilli is given. Which sign would indicate a positive test result? a) An indurated wheal under 10 mm in diameter appears in 6 to 12 hours. b) A flat circumscribed area over 10 mm in diameter appears in 48 to 72 hours. c) An indurated wheal over 10 mm in diameter appears in 48 to 72 hours. d) A flat, circumscribed area under 10 mm in diameter appears in 6 to 12 hours.

c) An indurated wheal over 10 mm in diameter appears in 48 to 72 hours. Rationale: A positive PPD result would be an indurated wheal over 10 mm in diameter that appears in 48 to 72 hours. The area must be a raised wheal, not a flat, circumscribed area.

When completing the morning postpartum data collection, the nurse notices the client's perineal pad is completely saturated. Which action should be the nurse's first response? a) Vigorously massage the fundus. b) Immediately call the primary care provider. c) Ask the client when she last changed her perineal pad. d) Have the charge nurse review the assessment.

c) Ask the client when she last changed her perineal pad. Rationale: If the morning assessment is done relatively yearly, it's possible that the client hasn't yet been to the bathroom, in which case her perineal pad may have been in place all night. Secondly, her lochia may have pooled during the night, resulting in a heavy flow in the morning. Vigorous massage of the fundus, which is indicated for a boggy uterus, wouldn't be recommended as a first response until the client had gone to the bathroom, changed her perineal pad, and emptied her bladder. The nurse wouldn't want to call the primary care provider unnecessarily. If the nurse were uncertain, it would be appropriate to have another qualified individual check the client but only after a complete assessment of the client's status.

A pregnant woman is admitted to the hospital with a diagnosis of placenta previa. Which of the following would be the priority for this woman on admission? a) Performing a vaginal examination to assess the extent of bleeding b) Helping the woman remain ambulatory to reduce bleeding c) Assessing fetal heart tones by use of an external monitor d) Assessing uterine contractions by an internal pressure gauge

c) Assessing fetal heart tones by use of an external monitor Rationale: Not disrupting the placenta is a prime responsibility. An internal monitor, a vaginal examination, and remaining ambulatory could all do this and thus are contraindicated.

Which of the following would the nurse interpret as indicating that a pregnant client with gestational hypertension has developed severe preeclampsia? a) Proteinuria of 300 mg/24 hours b) Blood pressure of 150/100 mm Hg c) Blurred vision d) Mild facial edema

c) Blurred vision Rationale: Visual symptoms such as blurred vision and blind spots suggest severe preeclampsia. Severe preeclampsia is characterized by a blood pressure of 160/110 mm Hg. Mild facial edema or hand edema occurs with mild preeclampsia. Proteinuria in severe preeclampsia is greater than 500 mg/24 hours.

A new mother has been reluctant to hold her newborn. A nurse can promote this mother's attachment to her newborn by a) Showing a video of parents feeding their babies b) Talking about how the nurse held her own newborn while on the delivery table c) Bringing the newborn into the room d) Allowing the mother to pick the best time to hold her newborn

c) Bringing the newborn into the room Rationale: Proximity of the newborn and the mother can promote interest in the newborn and a desire to hold. Exposure to other mothers and their behaviors can only serve to set up unrealistic and fearful situations for a reluctant mother.

A woman with a positive history of genital herpes is in active labor. She has small pin-point vesicles in the perineum area. Her membranes are ruptured, she is dilated 5cm, effaced 70%. The nurse should anticipate what type of delivery? a) Forceps assisted. b) Vacuum assisted. c) Cesarean. d) Spontaneous vaginal.

c) Cesarean. Rationale: An active herpes infection can be passed to the fetus during labor or with ruptured amniotic membranes. The nurse should anticipate the infant will be delivered via a cesarean birth. The risk of transmitting herpes to the baby would be increased if the baby were born by spontaneous vaginal delivery, vacuum assisted delivery, or forceps assisted delivery.

A client has been admitted to the hospital with a diagnosis of severe pre-eclampsia. Which of the following is the priority nursing? a) Check for vaginal bleeding every 15 minutes b) Keep the client on her side so that secretions can drain from her mouth c) Confine the client to bed rest in a darkened room d) Administer oxygen by face mask

c) Confine the client to bed rest in a darkened room Rationale: With severe pre-eclampsia, most women are hospitalized so that bed rest can be enforced and a woman can be observed more closely than she can be on home care. Darken the room if possible because a bright light can also trigger seizures. The other interventions listed pertain to a client who has experienced a seizure and has thus progressed to eclampsia.

When administering magnesium sulfate to a woman with severe preeclampsia, which finding would alert the nurse to the development of magnesium toxicity? a) Seizures b) Elevated liver enzymes c) Diminished reflexes d) Serum magnesium level of 6.5 mEq/L

c) Diminished reflexes Rationale: Diminished or absent reflexes occur when a client develops magnesium toxicity. Elevated liver enzymes are unrelated to magnesium toxicity and may indicate the development of HELLP syndrome. The onset of seizure activity indicates eclampsia. A serum magnesium level of 6.5 mEq/L would fall within the therapeutic range of 4 to 7 mEq/L.

Elevation of a patient's temperature is a crucial first sign of infection. However, when is elevated temperature not a warning sign of impending infection? a) When the white blood cell count is less than 10,000/mm³ b) After any period of decreased intake c) During the first 24 hours after delivery owing to dehydration from exertion d) When the elevated temperature exceeds 100.4° F

c) During the first 24 hours after delivery owing to dehydration from exertion Rationale: Rapid breathing during labor and delivery and limited oral intake can cause a self-limited period of dehydration that is resolved after delivery by the diuresis that shortly follows. The option of "any period" is too broad and falsely encompasses all conditions. The other options are signs of infection.

Nursing care for women diagnosed with gestational diabetes includes which of the following? a) Education for the women on life-long diabetic needs b) Counseling the patient on the need for cesarean birth c) Encourage blood glucose control d) Referral for the infant to diabetic care after delivery

c) Encourage blood glucose control Rationale: The patient will need to have control of her blood sugar during her pregnancy to decrease any complications associated with gestational diabetes. The patient with gestational diabetes does not need to be counseled on the need to have a cesarean delivery, receive education on life-long diabetic needs, or have the infant referred for diabetic care after delivery.

A client who gave birth by cesarean delivery 3 days ago is bottle-feeding her neonate. While collecting data the nurse notes that vital signs are stable, the fundus is four fingerbreadths below the umbilicus, lochia are small and red, and the client reports discomfort in her breasts, which are hard and warm to touch. The best nursing intervention based on this data would be: a) Having the client stand facing in a warm shower. b) Using a breast pump to facilitate removal of stagnant breast milk. c) Encouraging the client to wear a supportive bra. d) Informing the physician that the client is showing early signs of breast infection.

c) Encouraging the client to wear a supportive bra. Rationale: These assessment findings are normal for the third postpartum day. Hard, warm breasts indicate engorgement, which occurs approximately 3 days after birth. Vital signs are stable and don't indicate signs of infection. The client should be encouraged to wear a supportive bra, which will help minimize engorgement and decrease nipple stimulation. Ice packs can reduce vasocongestion and relieve discomfort. Warm water and a breast pump will stimulate milk production.

Which of the following is an appropriate nursing intervention for prevention of a urinary tract infection (UTI) in the postpartum woman? a) Increasing intravenous fluids. b) Increasing oral fluid intake. c) Encouraging the woman to empty her bladder completely every 2 to 4 hours. d) Screening for bacteriuria in the urine.

c) Encouraging the woman to empty her bladder completely every 2 to 4 hours. Rationale: The nurse should advise the woman to urinate every 2 to 4 hours while awake to prevent overdistention and trauma to the bladder. Maintaining a good fluid intake is also important, but it is not necessary to increase fluids if the woman is consuming enough. Screening for bacteria in the urine would require a physician's order and is not necessary as a prevention measure.

A pregnant woman is diagnosed with abruptio placentae. When reviewing the woman's medical record, which of the following would the nurse expect to find? a) Bright red vaginal bleeding b) Fetal heart rate within normal range c) Firm, rigid uterus on palpation d) Absence of pain

c) Firm, rigid uterus on palpation Rationale: The uterus is firm to rigid to the touch with abruptio placentae; it is soft and relaxed with placenta previa. Bleeding associated with abruptio placentae occurs suddenly and is usually dark in color. Bleeding also may not be visible. Bright red vaginal bleeding is associated with placenta previa. Fetal distress or absent fetal heart rate may be noted with abruptio placentae. The woman with abruptio placentae usually experiences constant uterine tenderness on palpation.

A woman yesterday delivered a child with a cleft palate. The newborn is in the special care nursery, and the mother has seen the newborn only at delivery. The nurse's priority is to assist the mother to a) Visit the child in the nursery b) Review causes of a cleft palate c) Grieve for the loss of the perfect baby d) Care for herself

c) Grieve for the loss of the perfect baby Rationale: Grief is the response to loss. The process of mourning will take precedence over the mother's self-care in this initial period. The nurse will assess the mother to note her physical condition, but the mother will be focused on the child. The mother can be assisted to determine the appropriate time to see the child, and then attachment can be promoted.

Human papillomavirus (HPV) can cause condylomata acuminata that can develop in clusters on the vulva, within the vagina, on the cervix, or around the anus. What is their risk? a) Neonatal auricular papillomas b) Blockage of the birth canal c) Heavy bleeding during vaginal delivery d) Neonatal hemorrhage

c) Heavy bleeding during vaginal delivery Rationale: Genital warts have a tendency to increase in size during pregnancy. This may result in heavy bleeding during vaginal delivery. The pregnant woman can pass HPV to her fetus during the birth process. In rare instances, neonatal HPV infection can result in life-threatening laryngeal papillomas. HPV infection transmitted to the infant may not appear for as long as 10 years after birth.

For which of the following problems would the nurse be alert in a pregnant woman with gestational diabetes? a) Placenta previa related to diabetes mellitus b) Hypotension related to glucose/insulin imbalance c) Hydramnios related to glucose/insulin imbalance d) Cerebral vascular accident related to diabetes mellitus

c) Hydramnios related to glucose/insulin imbalance Rationale: Hyperglycemia tends to lead to excessive amniotic fluid (hydramnios) because of osmotic pressure fluid shifts.

Which of the following changes in pregnancy would the nurse identify as a contributing factor for arterial thrombosis, especially for the woman with atrial fibrillation? a) Increased cardiac output b) Elevation of diaphragm c) Hypercoagulable state d) Increase in blood volume

c) Hypercoagulable state Rationale: The nurse should identify that the increased risk of arterial thrombosis in atrial fibrillation is due to hypercoagulable state of pregnancy. During pregnancy there is a state of hypercoagulation. This increases the risk of arterial thrombosis in clients having atrial fibrillation and artificial valves. Increased cardiac output and blood volume do not cause arterial thrombosis. Elevation of the diaphragm is due to the uterine distension and it causes a shift in the QRS axis and is not a associated with arterial thrombosis.

A woman is admitted with a diagnosis of ectopic pregnancy. For which of the following would you anticipate beginning preparation? a) Bed rest for the next 4 weeks. b) Intravenous administration of a tocolytic. c) Immediate surgery. d) Internal uterine monitoring.

c) Immediate surgery. Rationale: Ectopic pregnancy means an embryo has implanted outside the uterus, usually in the fallopian tube. Surgery is usually necessary to remove the growing structure before the tube ruptures or repair the tube if rupture has already occurred. Bed rest will not correct the problem of an ectopic pregnancy. Administering a tocolytic is not indicated, nor is internal uterine monitoring. This makes options A, B, and D incorrect

A woman who delivered 10 hours ago is ambulating to the bathroom and calls for assistance with perineal care. When the nurse touches her skin, he notices that she is excessively warm. After reinforcing the woman's self-care, the nurse encourages increased oral intake. Why was this the appropriate instruction to give to this patient? a) Increased intake will increase the patient's output and therefore will provide an opportunity for more frequent perineal self-care. b) The patient needs to walk to the bathroom more often. c) Increased intake will rehydrate the patient and decrease her skin temperature. d) The patient will have to call for the nurse's help more often.

c) Increased intake will rehydrate the patient and decrease her skin temperature. Rationale: The perception of increased skin temperature a short time post delivery is related to dehydration from the exertion of labor. Therefore rehydration should help to decrease skin temperature. Information is insufficient to suggest the presence of infection. Goals of more frequent perineal care and ambulation, as well as reinforcement of patient teaching, are not appropriate in this situation.

Which of the following changes in B/P assessment findings during the second trimester indicate the highest risk for preeclampsia. a) Initial BP 120/80, current BP 130/88. b) Initial BP 110/60, current BP 112/86. c) Initial BP 100/70, current BP 140/90. d) Initial BP 140/85, current BP 130/80.

c) Initial BP 100/70, current BP 140/90. Rationale: A rise in blood pressure to above 140/90 is a concern the patient may be developing preeclampsia. The B/Ps noted in options A, C, and D are not indicative of developing preeclampsia, so these are incorrect responses.

Which of the following would the nurse include in the plan of care for a woman with premature rupture of membranes if her fetus's lungs are mature? a) Administration of corticosteroids b) Observation for signs of infection c) Labor induction d) Reduction in physical activity level

c) Labor induction Rationale: With premature rupture of membranes in a woman whose fetus has mature lungs, induction of labor is initiated. Reducing physical activity, observing for signs of infection, and giving corticosteroids may be used for the woman with PROM when the fetal lungs are immature.

A client in her eighth month of pregnancy who has cardiac disease is experiencing profound shortness of breath and a cough that produces blood-speckled sputum, in addition to systemic hypotension. The nurse recognizes that this patient most likely is experiencing which of the following conditions? a) Peripartal cardiomyopathy b) Pulmonary embolism c) Left-sided heart failure d) Right-sided heart failure

c) Left-sided heart failure Rationale: In left-sided heart failure, the left ventricle cannot move the large volume of blood forward that it has received by the left atrium from the pulmonary circulation. It is characterized by a decrease in systemic blood pressure and pulmonary edema that produces profound shortness of breath. If pulmonary capillaries rupture under the pressure, small amounts of blood leak into the alveoli and the woman develops a productive cough with blood-speckled sputum. Right-sided heart failure is characterized by extreme liver enlargement, distention of abdominal and lower extremity vessels, ascites, and peripheral edema. A woman with peripartal cardiomyopathy develops signs of myocardial failure such as shortness of breath, chest pain, and nondependent edema. Her heart increases in size (cardiomegaly).The signs of a pulmonary embolism include chest pain, a sudden onset of dyspnea, a cough with hemoptysis, tachycardia or missed beats, or dizziness and fainting.

Which of the following changes in insulin is most likely to occur in a woman during pregnancy? a) Unavailable because it is used by the fetus b) Not released because of pressure on the pancreas c) Less effective than normal d) Enhanced secretion from normal

c) Less effective than normal Rationale: Somatotropin released by the placenta makes insulin less effective. This is a safeguard against hypoglycemia.

A woman with Class II heart disease is in the third trimester of her pregnancy. She's been taking good care of herself and has had little difficulty, but to be on the safe side the obstetrician has ordered bed rest for her for the final month. For her own and the baby's safety, in what position should the nurse advise the patient to sleep? a) Lie flat on her back. b) Stay in high Fowler's position. c) Lie in a semirecumbent position. d) Use pillows and wedges to stay in a fully recumbent position.

c) Lie in a semirecumbent position. Rationale: Semirecumbent position is the best position for circulation of the mother and fetus. Laying flat on the back can induce supine hypotensive syndrome and fully recumbent impedes other circulation. Therefore options A, B, and D are incorrect answers for this question.

Which of the following would be the physiologic basis for a placenta previa? a) A loose placental implantation b) A placenta with multiple lobes c) Low placental implantation d) A uterus with a midseptum

c) Low placental implantation Rationale: The cause of placenta previa is usually unknown, but for some reason the placenta is implanted low instead of high on the uterus.

During a routine prenatal check-up, the nurse interviews a pregnant client to identify possible risk factors for developing gestational diabetes. Which of the following would alert the nurse to an increased risk? Select all that apply. a) Previous history of spontaneous abortion b) Younger maternal age at pregnancy c) Maternal obesity with body mass index more than 35 d) Previous birth of small for gestational age baby e) Client of African-American lineage

c) Maternal obesity with body mass index more than 35 d) Previous birth of small for gestational age baby e) Client of African-American lineage Rationale: The risk factors for gestational diabetes include previous history of spontaneous abortion, maternal obesity with body mass index (BMI) more than 35, and client of African-American lineage. The other risk factors for gestational diabetes are previous history of stillbirth, birth of large for gestational age infant and advancing maternal age. High-risk ethnic groups include African American, Hispanic and Native North American.

Which of the following would the nurse prepare to administer if ordered as treatment for an unruptured ectopic pregnancy? a) Promethazine b) Oxytocin c) Methotrexate d) Ondansetron

c) Methotrexate Rationale: Methotrexate, a folic acid antagonist that inhibits cell division in the developing embryo, is most commonly used to treat ectopic pregnancy. Oxytocin is used to stimulate uterine contractions and would be inappropriate for use with an ectopic pregnancy. Promethazine and ondansetron are antiemetics that may be used to treat hyperemesis gravidarum.

Your patient is pregnant and she has tested positive for cytomegalovirus. What can this cause in the newborn? a) Hypertension b) Clubbed fingers and toes c) Microcephaly d) Bicuspid valve stenosis

c) Microcephaly Rationale: Signs that are likely to be present in the 10 percent of newborns who are symptomatic at birth include microcephaly, seizures, IUGR, hepatosplenomegaly, jaundice, and rash.

A woman is 9 weeks gestation and admitted to the obstetrical unit for hyperemesis gravidarum. The highest priority intervention the nurse should anticipate is which of the following? a) Administration of antiemetics b) IV rehydration c) NPO for 24 hours d) Bed rest with bathroom privileges

c) NPO for 24 hours Rationale: The initial intervention to have priority is to stop all food and fluids by mouth until the vomiting has stopped. The next steps will depend on the severity of the hyperemesis gravidarum. Use of medications should be based on the severity of the hyperemesis gravidarum and only under orders as most medications are pregnancy category C. IV rehydration may be ordered by the physician.

Which of the following factors in a postpartum woman's history would lead the nurse to watch the woman closely for an infection? a) Multiparity b) Labor of 12 hours c) Placenta removed via manual extraction d) Hemoglobin of 11.5 mg/dL

c) Placenta removed via manual extraction Rationale: Manual removal of the placenta, a labor longer than 24 hours, a hemoglobin less than 10.5 mg/dL, and multiparity, such as more than three births closely spaced together, would place the woman at risk for postpartum hemorrhage.

A woman in labor has sharp fundal pain accompanied by slight vaginal bleeding. Which of the following would be the most likely cause of these symptoms? a) Possible fetal death or injury. b) Placenta previa obstructing the cervix. c) Premature separation of the placenta. d) Preterm labor that was undiagnosed.

c) Premature separation of the placenta. Rationale: Premature separation of the placenta begins with sharp fundal pain, usually followed by vaginal bleeding. Placenta previa usually produces painless bleeding; Preterm labor contractions are more often described as cramping. Possible fetal death or injury does not present with sharp fundal pain. It is usually painless.

A patient delivered 2 days ago and is preparing for discharge. The nurse assesses respirations to be 26 rpm and labored, and the patient was short of breath ambulating from the bathroom this morning. Lung sounds are clear. The nurse alerts the physician and the nurse-midwife to her concern that the patient may be experiencing a) Upper respiratory infection b) Thrombophlebitis c) Pulmonary embolism d) Mitral valve collapse

c) Pulmonary embolism Rationale: These symptoms suggest a pulmonary embolism. Mitral valve collapse and thrombophlebitis would not present with these symptoms; infection would have a febrile response with changes in lung sounds.

When assessing a woman with an ectopic pregnancy, which of the following would lead the nurse to suspect that the tube has ruptured? a) Breast tenderness b) Vaginal spotting c) Referred shoulder pain d) Nausea

c) Referred shoulder pain Rationale: Referred pain to the shoulder area indicates bleeding into the abdomen caused by phrenic nerve irritation when a tubal pregnancy ruptures. Vaginal spotting, nausea, and breast tenderness are typical findings of early pregnancy and an unruptured ectopic pregnancy.

Not all mothers express joy at seeing their newborn upon delivery and during their hospitalization. A behavior that indicates impaired attachment of the mother to the newborn is a) Dressing the child in old clothes b) Giving the child an uncommon name c) Referring to a facial feature as "ugly" d) Bottle feeding

c) Referring to a facial feature as "ugly" Rationale: Making negative comments about a newborn's features is a warning sign of impending attachment difficulties. The other options may be culturally rooted.

Which measure would be most effective in preventing isoimmunization during pregnancy? a) Cerclage b) Blood typing of mothers with type A or B blood c) RhoGAM administration to Rh-negative women d) Amniocentesis

c) RhoGAM administration to Rh-negative women Rationale: Rh incompatibility can be prevented with the use of RhoGAM. Hemolysis associated with ABO incompatibility is limited to mothers with type O blood and their fetuses with type A or B blood. Amniocentesis would be appropriate for treatment of polyhydramnios, not isoimmunization. Cerclage is a treatment for cervical insufficiency.

You are doing patient teaching with a 28 weeks' gestation woman who has tested positive for gestational diabetes mellitus (GDM). What would be important to include in your patient teaching? a) Her baby is at increased risk for neonatal diabetes mellitus. b) She is at increased risk for type I diabetes mellitus after her baby is born. c) She is at increased risk for type II diabetes mellitus after her baby is born. d) Her baby is at increased risk for type I diabetes mellitus.

c) She is at increased risk for type II diabetes mellitus after her baby is born. Rationale: The woman who develops GDM is at increased risk for developing type 2 DM after pregnancy.

A patient who has just delivered a baby girl demonstrates behavior not indicative of bonding when she does which of the following? a) Kisses the infant on her cheek b) Strokes the infants' head c) Talks to company and ignores the baby lying next to her d) Holds and smiles at the infant

c) Talks to company and ignores the baby lying next to her Rationale: Bonding is the close emotional attraction to a newborn by the parents that develops during the first 30 to 60 minutes after birth. The mother initiates bonding when she caresses her infant and exhibits certain behaviors typical of a mother tending to her child. Ignoring the infant while talking to visitors is not an example of proper bonding.

A 45-year-old pregnant woman with type O blood has had an amniocentesis to rule out Down syndrome. The fetus has type AB blood. What can the nurse warn the patient is a likely outcome if some fetal blood mixed with maternal blood during the procedure? a) The baby will develop hemolytic anemia. b) Preterm delivery c) The baby will have postdelivery jaundice. d) Placental abruption

c) The baby will have postdelivery jaundice. Rationale: The infant and mother have ABO incompatibility. The result is a development of antibodies and breaking down of the blood resulting in jaundice in the infant after delivery. The mixing of some fetal blood with maternal blood during the amniocentesis would not cause placental abruption or preterm delivery. Hemolytic anemia is caused by Rh incompatibility, not ABO incompatibility.

You help a postpartum woman out of bed for the first time postpartally and notice that she has a very heavy lochia flow. Which of the following assessment findings would best help you decide that the flow is within normal limits? a) The flow contains large clots. b) The flow is over 500 mL. c) The color of the flow is red. d) Her uterus is soft to your touch.

c) The color of the flow is red. Rationale: A typical lochia flow on the first day postpartally is red; it contains no large clots; the uterus is firm, indicating that it is well contracted.

A woman in week 16 of her pregnancy calls her physician's office to report that she has experienced abdominal cramping, cervical dilation, vaginal spotting, and the passing of tissue. The nurse instructs the client to bring the passed tissue to the hospital with her. What is the correct rationale for this instruction? a) To determine the stage of development of the fetus b) To determine whether infection is present c) To determine whether gestational trophoblastic disease is present d) To determine whether the fetus is viable

c) To determine whether gestational trophoblastic disease is present Rationale: Gestational trophoblastic disease is abnormal proliferation and then degeneration of the trophoblastic villi. The embryo fails to develop beyond a primitive start. Abnormal trophoblast cells must be identified because they are associated with choriocarcinoma, a rapidly metastasizing malignancy. This is why it is important for any woman who begins to miscarry at home to bring any clots or tissue passed to the hospital with her. The presence of clear fluid-filled cysts changes the diagnosis from a simple miscarriage to gestational trophoblastic disease. The client is not instructed to bring in passed tissue to determine whether infection is present or the fetus is viable or to determine the stage of development of the fetus.

A young mother delivers twin boys who shared the same placenta. What serious complication are they at risk for? a) TORCH syndrome b) ABO incompatability c) Twin-to-twin transfusion syndrome (TTTS) d) HELLP syndrome

c) Twin-to-twin transfusion syndrome (TTTS) Rationale: When twins share a placenta, a serious condition called twin-to-twin transfusion syndrome (TTTS) can occur.

A woman at 8 weeks' gestation is admitted for ectopic pregnancy. She is asking why this has occurred. The nurse knows that which of the following is a known risk factor for ectopic pregnancy? a) High number of pregnancies b) Multiple gestation pregnancy c) Use of IUD for contraception d) Use of oral contraceptives

c) Use of IUD for contraception Rationale: Use of an IUD with progesterone has a known increased risk for development of ectopic pregnancies. The nurse needs to complete a full history of the patient to determine if she had any other risk factors for an ectopic pregnancy. Adhesions, scarring, and narrowing of the tubal lumen may block the zygote's progress to the uterus. Any condition or surgical procedure that can injure a fallopian tube increases the risk. Examples include salpingitis, infection of the fallopian tube, endometriosis, history of prior ectopic pregnancy, any type of tubal surgery, congenital malformation of the tube, and multiple elective abortions. Conditions that inhibit peristalsis of the tube can result in tubal pregnancy. Hormonal factors may play a role because tubal pregnancy occurs more frequently in women who take fertility drugs or who use progesterone intrauterine contraceptive devices (IUDs). A high number of pregnancies, multiple gestation pregnancy, and the use of oral contraceptives are not known risk factors for ectopic pregnancy.

A postpartum client who had a cesarean birth reports right calf pain to the nurse. The nurse observes that the client has nonpitting edema from her right knee to her foot. The nurse knows to prepare the client for which test first? a) Transthoracic echocardiogram b) Noninvasive arterial studies of the right leg c) Venous duplex ultrasound of the right leg d) Venogram of the right leg

c) Venous duplex ultrasound of the right leg Rationale: Right calf pain and nonpitting edema may indicate deep vein thrombosis (DVT). Postpartum clients and clients who have had abdominal surgery are at increased risk for DVT. Venous duplex ultrasound is a noninvasive test that visualizes the veins and assesses blood flow patterns. A venogram is an invasive test that utilizes dye and radiation to create images of the veins and wouldn't be the first choice. Transthoracic echocardiography looks at cardiac structures and isn't indicated at this time. Right calf pain and edema are symptoms of venous outflow obstruction, not arterial insufficiency.

A woman states that she still feels exhausted on her second postpartal day. Your best advice for her would be to do which of the following? a) Avoid elevating her feet when she rests in a chair. b) Walk the length of the hallway to regain her strength. c) Walk with you the length of her room. d) Avoid getting out of bed for another 2 days.

c) Walk with you the length of her room. Rationale: Most women report feeling exhausted following childbirth. Ambulation is important, however, so a small amount, such as walking across a room, should be encouraged.

A postpartal woman asks you about perineal care. Which of the following recommendations would you give? a) Use an alcohol wipe to wash her suture line. b) Avoid using soap in her perineal care. c) Wash her perineum with her daily shower. d) Refrain from washing lochia from the suture line.

c) Wash her perineum with her daily shower. Rationale: A suture line should be kept free of lochia to discourage infection. Washing with soap and water at the time of a shower will help to do this.

A pregnant patient with mitral stenosis needs to begin taking an anticoagulant. The nurse identifies the drug of choice, which is used in early pregnancy and again during the last month of pregnancy, to be which of the following? a) aspirin b) coumadin c) heparin d) levonox

c) heparin Rationale: If an anticoagulant is required, heparin is the drug of choice for the beginning and the end of pregnancy. Heparin does not cross the placenta barrier.

A pregnant woman diagnosed with cardiac disease 4 years ago is told that her pregnancy is a high-risk pregnancy. The nurse then explains that the danger occurs primarily because of the increase in circulatory volume. The nurse informs the patient that the most dangerous time for her is when? a) in weeks 12-20 b) in weeks 8-12 c) in weeks 28-32 d) in weeks 20-28

c) in weeks 28-32 Rationale: The danger of pregnancy in a woman with cardiac disease occurs primarily because of the increase in circulatory volume. The most dangerous time for a woman is in weeks 28 to 32, just after the blood volume peaks.

A new mother talking to a friend states, "I wish my baby was more like yours. You are so lucky. My baby has not slept straight through the night even once. It seems like all she wants to do is breastfeed. I am so tired of her." This is an example of which of the following? a) positive bonding b) negative bonding c) negative attachment d) positive attachment

c) negative attachment Rationale: Expressing disappointment or displeasure in the infant, failing to explore the infant visually or physically, and failing to claim the infant as part of the family are just a few examples of negative attachment behaviors.

A nursing instructor is teaching students about pre-existing illnesses and how they can complicate a pregnancy. The instructor recognizes a need for further education when one of the students makes which statement? a) "A pregnant woman needs to be careful of and cautious about accidents and illnesses during her pregnancy." b) "A pregnant woman with a chronic illness can put the fetus at risk." c) "A pregnant woman with a chronic condition can put herself at risk." d) "A pregnant woman does not have to worry about contracting new illnesses during pregnancy."

d) "A pregnant woman does not have to worry about contracting new illnesses during pregnancy." Rationale: When a woman enters a pregnancy with a chronic illness, it can put both her and the fetus at risk. She needs to be cautious about developing a new illness during her pregnancy as well as having an accident during the pregnancy.

A young patient with a cardiac problem wants to get pregnant and tells the nurse that she is sad that she will never be able to have a baby. What is the best response by the nurse? a) "Cardiovascular problems are not a concern during pregnancy." b) "Women with your problem should never get pregnant, because the risks and dangers are too high for you and the fetus." c) "If you get pregnant, you are likely to face many complications." d) "Because of improved management, more women with cardiac problems can complete pregnancies successfully."

d) "Because of improved management, more women with cardiac problems can complete pregnancies successfully." Rationale: Because of improved management of cardiac disease, women who might never have risked pregnancy in the past can complete pregnancies successfully today.

Which of the following is the best question the nurse can ask a woman who is leaving the hospital after experiencing a complete spontaneous abortion? a) "Did you know that 75 percent of women who are trying to get pregnant experience spontaneous abortions like you have?" b) "Are you going to try again?" c) "May I give you some resources that you can use to try to stop smoking?" d) "Do you have someone to talk to or may I give you the names and numbers for some possible grief counselors?"

d) "Do you have someone to talk to or may I give you the names and numbers for some possible grief counselors?" Rationale: When a woman has a spontaneous abortion one important consideration is the emotional needs of the woman once she is home. She may not want to talk about the loss for a period of time, but the nurse needs to determine her support system for the future. Asking the woman if she is "going to try again" is an inappropriate question for the nurse to ask, and diminishes the experience of having a spontaneous abortion. Giving the woman statistical information on spontaneous abortions is not appropriate when this patient needs support and caring concern. Offering to give the patient resources to aid in smoking cessation is not addressed in the scenario so this is an inappropriate response.

The nurse is helping an indigent HIV-positive pregnant patient set up a postdelivery care plan for her baby. What is an appropriate question/statement during that discussion? a) "You understand that you can't breast-feed, right? Even though formula's expensive, you'll need to figure out a way to get it." b) "You're not planning to breast-feed are you? That would be dangerous for the baby." c) "HIV can be passed to the baby from breast-feeding so it's important that you give the baby formula. You probably can't afford formula can you?" d) "HIV can be passed to the baby from breast-feeding so it's important that you give the baby formula. Formula's pretty expensive so I'll give you some information for places you can contact if you ever need some help getting it."

d) "HIV can be passed to the baby from breast-feeding so it's important that you give the baby formula. Formula's pretty expensive so I'll give you some information for places you can contact if you ever need some help getting it." Rationale: HIV is possible to transmit via breastfeeding and formula is the only option for feeding. The nurse needs to provide positive information and offer to make referral or get assistance for the patient in financial need. Assuming the patient understands the reason she cannot breastfeed is not adequate nursing care. The patient needs the nurse to explain to her the reason for not breastfeeding her infant. Option B is incorrect as it does not fully answer the question being asked. Option C is inappropriate because telling the patient that she probably can't get formula since it is expensive is inappropriate. It is a negative comment about the patient and her status in life.

A nurse is talking to a newly pregnant woman who had a mitral valve replacement in the past. Which of the following statements by the patient reveals an understanding about the pre-existing condition? a) "I know I will be fine, but I worry about the fetus." b) "I know my baby will be fine, but I am worried about having a personal complication." c) "I don't have to worry about this, because I had the problem fixed before I became pregnant." d) "I understand that my fetus and I both are at risk for complications."

d) "I understand that my fetus and I both are at risk for complications." Rationale: When a woman enters pregnancy with a pre-existing condition, both she and her fetus can be at risk of developing complications.

Two days after giving birth, a client is to receive RhoGAM. The client asks the nurse why this is necessary. The most appropriate response from the nurse is: a) "RhoGAM suppresses antibody formation in a woman with Rh-positive blood who gave birth to a baby with Rh-negative blood." b) "RhoGAM suppresses antibody formation in a woman with Rh-positive blood who gave birth to a baby with Rh-positive blood." c) "RhoGAM suppresses antibody formation in a woman with Rh-negative blood who gave birth to a baby with Rh-negative blood." d) "RhoGAM suppresses antibody formation in a woman with Rh-negative blood who gave birth to a baby with Rh-positive blood."

d) "RhoGAM suppresses antibody formation in a woman with Rh-negative blood who gave birth to a baby with Rh-positive blood." Rationale: RhoGAM is indicated to suppress antibody formation in women with Rh-negative blood who gave birth to babies with Rh-positive blood. RhoGAM is also given to women with Rh-negative blood after miscarriage/pregnancy termination, abdominal trauma, ectopic pregnancy, and amniocentesis.

A woman who is breast-feeding her newborn says, "He doesn't seem to want to nurse. I must be doing something wrong." Which response by the nurse would be least helpful? a) "Breast-feeding takes time. Let's see what's happening." b) "Let me contact our lactation specialist and together maybe we can work through this." c) "Some babies latch on and catch on quickly; others take a little more time." d) "Some women just can't breast-feed. Maybe you're one of these women."

d) "Some women just can't breast-feed. Maybe you're one of these women." Rationale: This response ignores the woman's feelings and displays a negative attitude, indicating that the woman is at fault for the current situation. The woman needs reassurance that she can breast-feed and accomplish the task. She needs to understand that although breast-feeding is a natural process, it takes time and practice. By offering to observe her breast-feeding, the nurse offers support and can provide the woman with some practical suggestions as necessary. The statement that some babies need more time would reduce her frustration and uncertainty about her ability to breast-feed. A lactation consultant can provide the woman with additional support and teaching to foster empowerment in this situation.

Rhogam (Rh immune globulin) will be ordered for an RH - mother undergoing which of the following tests? a) Biophysical profile b) Contraction test c) Non-stress test d) Amniocentesis

d) Amniocentesis Rationale: Amniocentesis is a procedure requiring a needle to enter into the amniotic sac. There is a risk of mixing of the fetal and maternal blood which could result in blood incompatibility. A contraction test, a non-stress test, and biophysical profile are not invasive, so there would be no indication for Rhogam to be administered.

A client in week 38 of her pregnancy arrives at the emergency room reporting a sharp pain between her umbilicus and the iliac crest in her lower right abdomen that is increasing. She reports having experienced intense nausea and vomiting for the past 3 hours. Given these symptoms, the nurse suspects which of the following conditions? a) Ectopic pregnancy in conjunction with morning sickness b) Pulmonary embolism c) Left-sided heart failure d) Appendicitis

d) Appendicitis Rationale: With appendicitis, the nausea and vomiting is much more intense than with morning sickness and the pain is sharp and localized at McBurney's point (a point halfway between the umbilicus and the iliac crest on the lower right abdomen). With a ruptured ectopic pregnancy, a woman may experience abdominal pain that is either diffuse or sharp, but it is less likely to occur precisely at McBurney's point. The symptoms described do not match those of pulmonary embolism or left-sided heart failure.

A postpartal woman has a history of thrombophlebitis. Which of the following would help you to determine if she is developing this postpartally? a) Palpate her feet for tingling or numbness. b) Ask her if she feels any warmth in her legs. c) Take her temperature every 4 hours. d) Assess for calf redness and edema.

d) Assess for calf redness and edema. Rationale: Calf redness and edema, especially at the ankle and along the tibia, suggest thrombophlebitis.

You are doing patient teaching with a 30-year-old gravida 1 who has sickle cell anemia. She is not currently in crisis. Providing education on which topic is the highest nursing priority? a) Control of pain b) Constipation prevention c) Iron-rich foods d) Avoidance of infection

d) Avoidance of infection Rationale: Prevention of crises, if possible, is the focus of treatment for the pregnant woman with sickle cell anemia. Maintaining adequate hydration, avoiding infection, getting adequate rest, and eating a balanced diet are all common-sense strategies that decrease the risk of a crisis.

A pregnant woman in her 39th week of pregnancy presents to the clinic with a vaginal infection. She tests positive for chlamydia. What would this make her infant at risk for? a) Neonatal laryngeal papillomas b) Deafness c) Chicken pox d) Blindness

d) Blindness Rationale: A pregnant woman who contracts chlamydia is at increased risk for spontaneous abortion (miscarriage), pre-term rupture of membranes, and pre-term labor. The postpartum woman is at higher risk for endometritis (Fletcher & Ball, 2006). The fetus can encounter bacteria in the vagina during the birth process. If this happens, the newborn can develop pneumonia or conjunctivitis that can lead to blindness.

A pregnant client in her 22nd week of gestation arrives at the healthcare facility with complaints of excessive vaginal bleeding and absence of fetal movements. She is diagnosed as having second trimester fetal loss. Which of the following would the nurse anticipate as the cause of second trimester fetal loss? a) Ectopic pregnancy b) Congenital malformations c) Placenta previa d) Cervical incompetence

d) Cervical incompetence Rationale: The nurse should identify cervical incompetence as the cause for second trimester fetal loss. Cervical incompetence is a condition where there is painless cervical dilatation and results in second trimester fetal loss or can progress to preterm premature rupture of membranes. Ectopic pregnancy, congenital malformations and placenta previa are not involved in causing second trimester fetal loss. Ectopic pregnancy usually leads to first trimester fetal loss. Placenta previa is a condition in which there is implantation of the placenta to the lower uterine segment. Congenital malformations result in first trimester fetal loss.

A pregnant woman's pulse fluctuates throughout pregnancy and the early postpartum period. When assessing a 1-day postpartum woman's pulse, what is the first action a nurse should take in response to a rate of 56 bpm? a) Ask the woman what she has had to eat today. b) Advise that the woman not get out of bed until the nurse returns with assistance. c) Do nothing, this is normal. d) Compare the pulse rate of 56 bpm with her pulse rate on the first prenatal care visit.

d) Compare the pulse rate of 56 bpm with her pulse rate on the first prenatal care visit. Rationale: During pregnancy, the distended uterus obstructs the amount of venous blood returning to the heart; after birth, to accommodate the increased blood volume returning to the heart, stroke volume increases. Increased stroke volume reduces the pulse rate to between 50 and 70 beats per minute. Be certain to compare a woman's pulse rate with the slower range expected in the postpartum period, not with the normal pulse rate in the general population. Pulse usually stabilizes to prepregnancy levels within 10 days.

When providing nutritional counseling to a pregnant woman with diabetes, the nurse would urge the client to obtain most of her calories from which of the following? a) Protein sources b) Unsaturated fats c) Saturated fats d) Complex carbohydrates

d) Complex carbohydrates Rationale: The pregnant woman with diabetes is encouraged to eat three meals a day plus three snacks, with 40% of calories derived from good-quality complex carbohydrates, 35% of calories from protein sources, and 35% of calories from unsaturated fats. The intake of saturated fats should be limited during pregnancy, just as they should be for any person to reduce the risk of heart disease.

A woman with cardiac disease is 32 weeks gestation and alerts the nurse she has been having spells of light-headedness and dizziness every few days. The nurse provides which of the following interventions as an option to the patient? a) Bed rest and bathroom privileges only until delivery. b) Increase fluids and take more vitamins. c) The patient needs to discuss induction of labor with the physician. d) Decrease activity and rest more often.

d) Decrease activity and rest more often. Rationale: If the patient is developing symptoms associated with her heart condition, the first intervention is to monitor activity levels, decrease activity and treat the symptoms. At 32 weeks gestation, the suggestion to induce labor is not appropriate and without knowledge of the type of heart condition one would not recommend increase of fluids or vitamins. Total bed rest may be required if the symptoms do not resolve with decreased activity. Therefore options B, C, and D are incorrect,

A 28-year-old woman presents in the emergency room with severe abdominal pain. She has not had a normal period for 2 months but she reports that that is not abnormal for her. She has a history of endometriosis. What might the nurse suggest to the physician as a possible cause of the patient's abdominal pain? a) Healthy pregnancy b) Placenta previa c) Molar pregnancy d) Ectopic pregnancy

d) Ectopic pregnancy Rationale: Ectopic pregnancy can present with severe unilateral abdominal pain. Given the history of the client , the amount of pain, the possibility of ectopic pregnancy needs to be considered. A healthy pregnancy would not present with severe abdominal pain unless the patient were term and she was in labor. With a molar pregnancy the woman typically presents between 8 to 16 weeks' gestation with complaints of painless (usually) brown to bright red vaginal bleeding. Placenta previa typically presents with painless, bright red bleeding that begins with no warning.

When doing a health assessment, at which of the following locations would you expect to palpate the fundus in a woman on the second postpartal day and how should it feel? a) Fundus two fingerbreadths above symphysis pubis and hard b) Fundus height 4 cm below umbilicus and midline c) Fundus 4 cm above symphysis pubis and firm d) Fundus two fingerbreadths below umbilicus and firm

d) Fundus two fingerbreadths below umbilicus and firm Rationale: A uterine fundus typically regresses at a rate of one fingerbreadth a day, so on the second day postpartum it would be two fingerbreadths under the umbilicus and would feel firm.

A pregnant client with sickle cell anemia is at an increased risk for having a sickle cell crisis during pregnancy. Aggressive management for a client experiencing a sickle cell crisis with severe pain includes which measure? a) Acetaminophen (Tylenol) for pain b) Antihypertensive drugs c) Diuretic drugs d) I.V. fluids

d) I.V. fluids Rationale: A sickle cell crisis during pregnancy is usually managed by exchange transfusion, oxygen, and I.V. fluids. Antihypertensive drugs usually aren't necessary. Diuretics wouldn't be used unless fluid overload resulted. The client usually needs a stronger analgesic than acetaminophen to control the pain of a crisis.

A woman is being admitted to your hospital unit for severe preeclampsia. When deciding on where to place her, which of the following areas would be most appropriate? a) Near the elevator so she can be transported quickly b) Near the nurse's station so she can be observed closely c) By the nursery so she can maintain hope she will have a child d) In the back hallway where there is a quiet, private room

d) In the back hallway where there is a quiet, private room Rationale: A sudden noise can trigger a seizure in a severely preeclamptic woman. Room placement, therefore, should not be near noise, such as the nursery, the elevator, or nurse's station.

A nurse working on the postpartum floor is mentoring a new graduate and instructs the new nurse to make sure that patients empty their bladders. A full bladder can lead to which of the following complications? a) Fluid volume overload b) Ruptured bladder c) Permanent urinary incontinence d) Increased lochia drainage

d) Increased lochia drainage Rationale: If the bladder is full in a postpartum mother, lochia drainage will be more than normal because the uterus cannot contract to suppress the bleeding. The other options do not happen if a woman has a distended bladder.

A nurse is assessing a postpartum client. Which of the following measures is appropriate? a) Wear sterile gloves when assessing the pad and perineum. b) Perform the examination as quickly as possible. c) Place the client in a supine position with her arms overhead for the examination of her breasts and fundus. d) Instruct the client to empty her bladder before the examination.

d) Instruct the client to empty her bladder before the examination. Rationale: An empty bladder facilitates examination of the fundus. The client should be supine with arms at her sides and her knees bent. The arms-overhead position is unnecessary. Clean gloves should be used when assessing the perineum; sterile gloves are not necessary. The postpartum examination should not be done quickly. The nurse can take this time to teach the client about the changes in her body after delivery.

When providing education to a teenage prenatal class, the nurse states that infants born to teenage mothers are more likely to have which of the following? a) Lower mortality rates b) Genetic problems c) Post-date delivery d) Low-birth weight

d) Low-birth weight Rationale: Infants born to adolescent mothers are more likely to have a low-birth weight and poor outcomes and higher mortality rates when compared to infants of older mothers. Infants born to teenage mothers are not more likely to have genetic problems; they are more likely to be born pre-term rather than post-date.

A woman develops gestational diabetes. Which of the following assessments should she make daily? a) Test her urine for protein with a chemical reagent strip. b) Measure her uterine height by hand-span distance. c) Measure her abdominal diameter with a tape measure. d) Measure serum for glucose level by a finger prick.

d) Measure serum for glucose level by a finger prick. Rationale: Assessing serum glucose reveals both hyperglycemia and hypoglycemia.

During a routine prenatal visit, a client is found to have proteinuria and a blood pressure rise to 140/90 mm Hg. The nurse recognizes that the client has which of the following conditions? a) Eclampsia b) Severe pre-eclampsia c) Gestational hypertension d) Mild pre-eclampsia

d) Mild pre-eclampsia Rationale: A woman is said to have gestational hypertension when she develops an elevated blood pressure (140/90 mm Hg) but has no proteinuria or edema. If a seizure from gestational hypertension occurs, a woman has eclampsia, but any status above gestational hypertension and below a point of seizures is pre-eclampsia. A woman is said to be mildly pre-eclamptic when she has proteinuria and a blood pressure rise to 140/90 mm Hg, taken on two occasions at least 6 hours apart. A woman has passed from mild to severe pre-eclampsia when her blood pressure rises to 160 mm Hg systolic and 110 mm Hg diastolic or above on at least two occasions 6 hours apart at bed rest (the position in which blood pressure is lowest) or her diastolic pressure is 30 mm Hg above her prepregnancy level. Marked proteinuria, 3+ or 4+ on a random urine sample or more than 5 g in a 24-hour sample, and extensive edema are also present. A woman has passed into eclampsia when cerebral edema is so acute a grand-mal seizure (tonic-clonic) or coma has occurred.

The nurse is observing a client who gave birth yesterday. Where should the nurse expect to find the top of the client's fundus? a) At the level of the umbilicus b) Below the symphysis pubis c) One fingerbreadth above the umbilicus d) One fingerbreadth below the umbilicus

d) One fingerbreadth below the umbilicus Rationale: After a client gives birth, the height of her fundus should decrease by approximately one fingerbreadth (1 cm) each day. By the end of the first postpartum day, the fundus should be one fingerbreadth below the umbilicus. Immediately after birth, the fundus may be above the umbilicus; 6 to 12 hours after birth, it should be at the level of the umbilicus; 10 days after birth, it should be below the symphysis pubis.

A 32-year-old gravida 3 para 2 at 36 weeks' gestation comes to the obstetric department reporting abdominal pain. Her blood pressure is 164/90 mm/Hg, her pulse is 100 beats per minute, and her respirations are 24 per minute. She is restless and slightly diaphoretic with a small amount of dark red vaginal bleeding. What assessment should the nurse make next? a) Obtain a voided urine specimen and determine blood type. b) Measure fundal height. c) Check deep tendon reflexes. d) Palpate the fundus and check fetal heart rate.

d) Palpate the fundus and check fetal heart rate. Rationale: The classic signs of abruption placentea are pain, dark red vaginal bleeding, a rigid, board-like abdomen, hypertonic labor, and fetal distress.

Which finding would indicate to the nurse that a postpartum woman is experiencing bladder distention? a) Lochia is less than usual b) Bladder is nonpalpable c) Uterus is firm d) Percussion reveals dullness

d) Percussion reveals dullness Rationale: A distended bladder is dull on percussion and can be palpated as a rounded mass. In addition, the uterus would be boggy and lochia would be more than usual.

A 40-year-old woman comes to the clinic complaining of having missed her period for two months. A pregnancy test is positive. What is she and her fetus at increased risk for? a) Post-term delivery b) Type I diabetes Mellituus c) Type II diabetes Mellitus d) Placental abnormalities

d) Placental abnormalities Rationale: A woman older than 35 years is more likely to conceive a child with chromosomal abnormalities, such as Down syndrome. She is also at higher risk for spontaneous abortion (miscarriage), preeclampsia-eclampsia, gestational diabetes, pre-term delivery, bleeding and placental abnormalities, and other intrapartum complications.

The nurse is providing education to women who had diabetes prior to pregnancy. The nurse is discussing pregnancy-related complications from diabetes. Which of the following is a potential complication? a) Post-term delivery b) Small for gestation age infant c) Hypotension of pregnancy d) Polyhydramnios

d) Polyhydramnios Rationale: Polyhydramnios is an increase, or excess, in amniotic fluid and is a pregnancy-related complication associated with diabetes in pregnancy. An infant who is small for gestational age is not associated with a mother who had diabetes prior to pregnancy. Other pregnancy-related complications associated with pregestational DM include hypertensive disorders, preterm delivery, and shoulder dystocia.

A woman is pregnant and has asthma. Her physician has told her to continue taking prednisone during pregnancy, but she is concerned the drug may be teratogenic. What advice would be best to give her regarding this? a) Prednisone is a teratogenic drug, but she may need it to control her asthma symptoms. b) You would recommend she omit the drug during pregnancy. c) She should half her dose during the first 3 months of pregnancy. d) Prednisone is considered safe in the doses prescribed by her physician.

d) Prednisone is considered safe in the doses prescribed by her physician. Rationale: Women should take no medication during pregnancy except that prescribed by their primary-care provider. Prednisone may be prescribed safely because, although it may be teratogenic in animal models, it does not appear to be teratogenic in humans.

Which of the following is recommended to prevent transmission of HIV to a newborn if the mother has AIDS? a) Avoid scalp electrodes for internal fetal monitoring. b) Admit infant to NICU after delivery. c) Perform amniotomy. d) Prepare for cesarean delivery.

d) Prepare for cesarean delivery. Rationale: When a patient is HIV positive, the method of delivery preferred is cesarean. This method has the lowest transmission rate for passage of the HIV infection to the infant. The nurse should educate the woman on the standard of care for delivery in an HIV or AIDS positive mother. Avoiding scalp electrodes for internal fetal monitoring, admitting the infant to NICU, and performing an amniotomy are not recommended methods for preventing transmission of HIV to a newborn.

A woman develops HELLP syndrome. During labor, which of the following orders would you question? a) Assess her blood pressure every 15 minutes. b) Assess the urine output every hour. c) Urge her to lie on her left side during labor. d) Prepare her for epidural anesthesia.

d) Prepare her for epidural anesthesia. Rationale: A consequence of the HELLP syndrome is poor blood coagulation. Epidural anesthesia is not recommended when blood coagulation is in doubt.

Which of the following would you emphasize in the teaching plan for a postpartal woman who is reluctant to begin taking warm sitz baths? a) The longer a sitz bath is continued, the more therapeutic it becomes. b) Sitz baths may lead to increased postpartal infection. c) Sitz baths cause perineal vasoconstriction and decreased bleeding. d) Sitz baths increase the blood supply to the perineal area.

d) Sitz baths increase the blood supply to the perineal area. Rationale: Sitz baths decrease pain and aid healing by increasing blood flow to the perineum.

When assessing a pregnant woman with vaginal bleeding, which finding would lead the nurse to suspect an inevitable abortion? a) No passage of fetal tissue b) Slight vaginal bleeding c) Closed cervical os d) Strong abdominal cramping

d) Strong abdominal cramping Rationale: Strong abdominal cramping is associated with an inevitable abortion. Slight vaginal bleeding early in pregnancy and a closed cervical os are associated with a threatened abortion. With an inevitable abortion, passage of the products of conception may occur. No fetal tissue is passed with a threatened abortion.

It has been 8 hours since a woman gave birth vaginally to a healthy newborn. When assessing the woman's fundus, the nurse would expect to find it at: a) Between the umbilicus and symphysis pubis b) 2 cm below the umbilicus c) 1 cm below the umbilicus d) The level of the umbilicus

d) The level of the umbilicus Rationale: Approximately 6 to 12 hours after birth, the fundus is usually at the level of the umbilicus. The fundus is between the umbilicus and symphysis pubis 1 to 2 hours after birth. The fundus typically is 1 cm below the umbilicus on the first postpartum day and 2 cm below the umbilicus on the second postpartum day.

In returning to the hospital floor after a weekend off, the nurse takes over care of a pregnant patient who is resting in a darkened room. The patient is receiving betamethasone and magnesium sulfate. What could the nurse deduce from those findings? a) The patient is suffering from eclampsia and the care team is attempting to prevent stroke and induce labor. b) The patient is suffering from mild preeclampsia and the care team is attempting to stabilize her and the baby before discharging her to home. c) The patient is suffering from hypertension and the care team is trying to lower her blood pressure so that she may return home until the baby is full term. d) The patient is suffering from severe preeclampsia and the care team is attempting to prevent advancement of the disorder to eclampsia; they are attempting to help the baby's lungs mature quickly so that they can deliver as soon as possible.

d) The patient is suffering from severe preeclampsia and the care team is attempting to prevent advancement of the disorder to eclampsia; they are attempting to help the baby's lungs mature quickly so that they can deliver as soon as possible. Rationale: The administration of magnesium sulfate is to relax the skeletal muscles and raise the threshold for a seizure. The administration of the betamethasone is to try and hasten the maturity of the fetus' lungs for delivery. This woman is in advanced preeclampsia and must be monitored for progression to eclampsia. The scenario described does not indicate a patient with hypertension who may be discharged home once the condition is under control. A woman in eclampsia would either be seizing or comatose, not resting in a quiet room. Symptoms of mild preeclampsia are limited to slightly elevated blood pressure and small amounts of protein in the urine. Betamethasone may be indicated at this time.

While the nurse is weighing a pregnant woman at a regularly scheduled OB visit, the patient complains of vaginal itching, a great deal of foamy yellow-green discharge, and pain during intercourse. She says this is her first pregnancy and she didn't know this was what happened. What can the nurse tell her? a) This is not normal for pregnancy but the doctor might test her for a simple yeast infection. If it is a yeast infection, it can be treated with a single-dose suppository that will not harm the fetus. Remind the patient that she should call immediately if she has any symptoms that don't seem normal to her. b) This is not normal for pregnancy; the doctor might test her for chlamydia. If it is chlamydia, she and her partner can be treated with a 7-day course of antibiotics. Remind the patient that she should call immediately if she has any symptoms that don't seem normal to her. c) This is not normal for pregnancy; the doctor might test her for gonorrhea. If it is gonorrhea, she and her partner will be treated with antibiotics; they might be treated with different medications because some antibiotics normally used to treat gonorrhea are damaging to the fetus. Remind the patient that she should call immediately if she has any symptoms that don't seem normal to her. d) This is not normal for pregnancy; the doctor might test her for trichomoniasis. If it is trichomoniasis, she can be treated with an oral dose of metronidazole. Remind the patient that she should call immediately if she has any symptoms that don't seem normal to her.

d) This is not normal for pregnancy; the doctor might test her for trichomoniasis. If it is trichomoniasis, she can be treated with an oral dose of metronidazole. Remind the patient that she should call immediately if she has any symptoms that don't seem normal to her. Rationale: Trichomoniasis is caused by a one-celled protozoa. The symptoms include large amounts of foamy, yellow-green vaginal discharge. Treatment is with metronidazole, her partner needs to be treated as well. A yeast infection presents with a cottage-cheese like discharge, so option A is incorrect. Chlamydia often has no symptoms. If the woman does experience symptoms, these may include vaginal discharge, abnormal vaginal bleeding, and abdominal or pelvic pain. Gonorrhea may have symptoms so mild that they go unnoticed in the woman. The woman who contracts gonorrhea may have vaginal bleeding during sexual intercourse, pain, and burning while urinating, and a yellow or bloody vaginal discharge.

When developing a plan of care for a pregnant woman who is HIV-positive, which of the following is essential? a) Helping her choose a newborn feeding method b) Preparing the woman for cesarean birth c) Educating her about family planning d) Using Standard Precautions

d) Using Standard Precautions Rationale: For the pregnant woman who is HIV-positive, nurses must always use Standard Precautions to reduce the risk of HIV transmission. Educating the woman about family planning methods is not as important as adhering to Standard Precautions. The decision about the mode of delivery is based on the woman's viral load, duration of ruptured membranes, progress of labor, and other clinical factors. Breast-feeding is contraindicated, so helping her choose a feeding method would be inappropriate.

Which factor puts a client on her first postpartum day at risk for hemorrhage? a) Moderate amount of lochia rubra b) Hemoglobin level of 12 g/dl c) Thrombophlebitis d) Uterine atony

d) Uterine atony Rationale: Loss of uterine tone places a client at higher risk for hemorrhage. Thrombophlebitis doesn't increase the risk of hemorrhage during the postpartum period. The hemoglobin level and lochia flow are within acceptable limits.

The nurse through assessment can best differentiate between placenta previa and abruptio placentae by which of the following signs and or symptoms. a) Bleeding amount and consistency. b) Low back pain. c) Shape of the abdomen. d) Uterine tone and contractions of the uterus.

d) Uterine tone and contractions of the uterus. Rationale: With placenta previa the bleeding is often bright red and painless, with abruptio placentae the bleeding is usually dark and painful. The uterus is firm and hard and painful with the abruption; the uterus is often soft and lacks tone with the previa. The contractions of the uterus, low back pain, or the shape of the abdomen do not help to distinguish between placenta previa and abruption placentae.

A woman who is Rh negative asks you how many children she will be able to have before Rh incompatibility causes them to die in utero. Your best response would be that a) she will have to ask her physician. b) no more than three children is recommended. c) only her next child will be affected. d) as long as she receives RhIG, there is no limit.

d) as long as she receives RhIG, there is no limit. Rationale: Because RhIG supplies passive antibodies, it prevents the woman from forming antibodies. Without antibodies that could affect the fetus, the woman could have as many children as she wants.

A patient who delivered twins 6 hours ago becomes restless and nervous. Her blood pressure falls from 130/80 to 96/50. Her pulse drops from 80 to 56. She was induced earlier in the day and experienced abruptio placentae. Based on this information, what postpartum complication would the nurse expect is happening? a) pulmonary emboli b) infection c) fluid volume overload d) hemorrhage

d) hemorrhage Rationale: Some risk factors for developing hemorrhage after delivery include precipitous labor, uterine atony, placenta previa and abruptio placentae, labor induction, operative procedures, retained placenta fragments, prolonged third stage of labor, multiparity, and uterine overdistention.

A new mother who is breastfeeding reports that her right breast is very hard, tender, and painful. Upon examination the nurse notices several nodules and the breast feels very warm to the touch. What do these findings indicate to the nurse? a) too much milk being retained b) normal findings in breastfeeding mothers c) an improperly positioned baby during feedings d) mastitis

d) mastitis Rationale: Engorged breasts are hard, tender, and taut. If the breasts have nodules, masses, or areas of warmth, they may have plugged ducts, which can lead to mastitis if not treated promptly.

A woman with no previous history of heart disease begins to have symptoms of myocardial failure a few weeks before the delivery of her first child. Findings include shortness of breath, chest pain, and edema, with her heart also showing enlargement. Which disease should the nurse suspect? a) pulmonary valve stenosis b) left sided heart failure c) mitral stenosis d) peripartal cardiomyopathy

d) peripartal cardiomyopathy Rationale: Peripartal cardiomyopathy can occur in pregnancy without any previous history of heart disease. Symptoms include shortness of breath, chest pain, and edema; also, the heart begins to increase. Treatment is with a diuretic, an antidysrhythmic agent, digitalis, low weight heparin, and bed rest.

A woman who had a cesarean delivery of twins 6 hours ago reports shortness of breath and pain in her right calf. What complication should the nurse expect? a) fluid volume overload b) infection c) hemorrhage d) pulmonay emboli

d) pulmonay emboli Rationale: One of the postpartum danger signs is calf pain with dorsiflexion of the foot. This would indicate a deep vein thrombosis. With the shortness of breath the patient might have a pulmonary emboli. This scenario would require immediate interventions to prevent the patient's death.


Ensembles d'études connexes

Developing Cardiorespiratory Fitness

View Set

SEC+ 601. Chapter 14: Incident Response

View Set

Statistics for Behavioral Sciences: Chapter 9

View Set

PRACTICE QUIZ: CH.5 - Public Policing and Private Security

View Set

Chapter 10: Beginning and Ending the Speech

View Set

今天几月几号?What's The Date Today? (PinYin)

View Set

OSHA 30 - Quiz - 7 Material Storage

View Set

Ch 5 Newton's Third Law of Motion

View Set

Chekhov Final Exam on Plays (REMEMBER CHARACTERS' NAMES!!!)

View Set

Outstanding (Incredible) Final Geography Quizlet 5

View Set